SlideShare a Scribd company logo
Test Banks For
Guyton and Hall Textbook of Medical Physiology
14th Edition by John E. Hall; Michael E. Hall,
Chapters 1 - 86
Guyton and Hall Textbook of Medical Physiology 14th Edition Test Bank by John E. Hall, Michael
E. Hall Table of Contents
Unit I. Introduction to Physiology: The Cell and General Physiology
Chapter 1. Functional Organization of the Human Body and Control of the “Internal
Environment”
Chapter 2. The Cell and Its Functions
Chapter 3. Genetic Control of Protein Synthesis, Cell Function, and Cell Reproduction
Unit II. Membrane Physiology, Nerve, and Muscle
Chapter 4. Transport of Substances Through Cell Membranes
Chapter 5. Membrane Potentials and Action Potentials
Chapter 6. Contraction of Skeletal Muscle
Chapter 7. Excitation of Skeletal Muscle: Neuromuscular Transmission and Excitation-
Contraction Coupling
Chapter 8. Excitation and Contraction of Smooth Muscle
Unit III. The Heart
Chapter 9. Cardiac Muscle; The Heart as a Pump and Function of the Heart Valves
Chapter 10. Rhythmical Excitation of the Heart
Chapter 11. Fundamentals of Electrocardiography
Chapter 12. Electrocardiographic Interpretation of Cardiac Muscle and Coronary
Blood Flow Abnormalities: Vectorial Analysis
Chapter 13. Cardiac Arrhythmias and Their Electrocardiographic Interpretation
Unit IV. The Circulation
Chapter 14. Overview of the Circulation: Pressure, Flow, and Resistance
Chapter 15. Vascular Distensibility and Functions of the Arterial and Venous Systems
Chapter 16. The Microcirculation and Lymphatic System: Capillary Fluid Exchange,
Interstitial Fluid, and Lymph Flow
Chapter 17. Local and Humoral Control of Tissue Blood Flow
Chapter 18. Nervous Regulation of the Circulation and Rapid Control of Arterial Pressure
Chapter 19. Role of the Kidneys in Long-Term Control of Arterial Pressure and in
Hypertension: The Integrated System for Arterial Pressure Regulation
Chapter 20. Cardiac Output, Venous Return, and Their Regulation
Chapter 21. Muscle Blood Flow and Cardiac Output During Exercise; the Coronary
Circulation and Ischemic Heart Disease
Chapter 22. Cardiac Failure
Chapter 23. Heart Valves and Heart Sounds; Valvular and Congenital Heart Defects
Chapter 24. Circulatory Shock and Its Treatment
Unit V. The Body Fluids and Kidneys
Chapter 25. Regulation of Body Fluid Compartments: Extracellular and Intracellular Fluids;
Edema
Chapter 26. The Urinary System: Functional Anatomy and Urine Formation by the Kidneys
Chapter 27. Glomerular Filtration, Renal Blood Flow, and Their Control
Chapter 28. Renal Tubular Reabsorption and Secretion
Chapter 29. Urine Concentration and Dilution; Regulation of Extracellular Fluid
Osmolarity and Sodium Concentration
Chapter 30. Renal Regulation of Potassium, Calcium, Phosphate, and Magnesium;
Integration of Renal Mechanisms for Control of Blood Volume and Extracellular Fluid Volume
Chapter 31. Acid–Base Regulation
Chapter 32. Diuretics and Kidney Diseases
Unit VI. Blood Cells, Immunity, and Blood Coagulation
Chapter 33. Red Blood Cells, Anemia, and Polycythemia
Chapter 34. Resistance of the Body to Infection: I. Leukocytes, Granulocytes, the
Monocyte- Macrophage System, and Inflammation
Chapter 35. Resistance of the Body to Infection: II. Immunity and Allergy
Chapter 36. Blood Types; Transfusion; and Tissue and Organ Transplantation
Chapter 37. Hemostasis and Blood Coagulation
Unit VII. Respiration
Chapter 38. Pulmonary Ventilation
Chapter 39. Pulmonary Circulation, Pulmonary Edema, and Pleural Fluid
Chapter 40. Principles of Gas Exchange; Diffusion of Oxygen and Carbon Dioxide
Through the Respiratory Membrane
Chapter 41. Transport of Oxygen and Carbon Dioxide in Blood and Tissue Fluids
Chapter 42. Regulation of Respiration
Chapter 43. Respiratory Insufficiency—Pathophysiology, Diagnosis, Oxygen Therapy
Unit VIII. Aviation, Space, and Deep-Sea Diving Physiology
Chapter 44. Aviation, High Altitude, and Space Physiology
Chapter 45. Physiology of Deep-Sea Diving and Other Hyperbaric Conditions
Unit IX. The Nervous System: A. General Principles and Sensory Physiology
Chapter 46. Organization of the Nervous System, Basic Functions of Synapses, and
Neurotransmitters
Chapter 47. Sensory Receptors, Neuronal Circuits for Processing Information
Chapter 48. Somatic Sensations: I. General Organization, Tactile and Position Senses
Chapter 49. Somatic Sensations: II. Pain, Headache, and Thermal Sensations
Unit X. The Nervous System: B. The Special Senses
Chapter 50. The Eye: I. Optics of Vision
Chapter 51. The Eye: II. Receptor and Neural Function of the Retina
Chapter 52. The Eye: III. Central Neurophysiology of Vision
Chapter 53. The Sense of Hearing
Chapter 54. The Chemical Senses—Taste and Smell
Unit XI. The Nervous System: C. Motor and Integrative Neurophysiology
Chapter 55. Spinal Cord Motor Functions; the Cord Reflexes
Chapter 56. Cortical and Brain Stem Control of Motor Function
Chapter 57. Cerebellum and Basal Ganglia Contributions to Overall Motor Control
Chapter 58. Cerebral Cortex, Intellectual Functions of the Brain, Learning, and Memory
Chapter 59. The Limbic System and the Hypothalamus—Behavioral and Motivational
Mechanisms of the Brain
Chapter 60. States of Brain Activity—Sleep, Brain Waves, Epilepsy, Psychoses, and Dementia
Chapter 61. The Autonomic Nervous System and the Adrenal Medulla
Chapter 62. Cerebral Blood Flow, Cerebrospinal Fluid, and Brain Metabolism
Unit XII. Gastrointestinal Physiology
Chapter 63. General Principles of Gastrointestinal Function—Motility, Nervous Control,
and Blood Circulation
Chapter 64. Propulsion and Mixing of Food in the Alimentary Tract
Chapter 65. Secretory Functions of the Alimentary Tract
Chapter 66. Digestion and Absorption in the Gastrointestinal Tract
Chapter 67. Physiology of Gastrointestinal Disorders
Unit XIII. Metabolism and Temperature Regulation
Chapter 68. Metabolism of Carbohydrates and Formation of Adenosine Triphosphate
Chapter 69. Lipid Metabolism
Chapter 70. Protein Metabolism
Chapter 71. The Liver
Chapter 72. Dietary Balances; Regulation of Feeding; Obesity and Starvation; Vitamins and
Minerals
Chapter 73. Energetics and Metabolic Rate
Chapter 74. Body Temperature Regulation and Fever
Unit XIV. Endocrinology and Reproduction
Chapter 75. Introduction to Endocrinology
Chapter 76. Pituitary Hormones and Their Control by the Hypothalamus
Chapter 77. Thyroid Metabolic Hormones
Chapter 78. Adrenocortical Hormones
Chapter 79. Insulin, Glucagon, and Diabetes Mellitus
Chapter 80. Parathyroid Hormone, Calcitonin, Calcium and Phosphate Metabolism, Vitamin
D, Bone, and Teeth
Chapter 81. Reproductive and Hormonal Functions of the Male (and Function of the Pineal
Gland)
Chapter 82. Female Physiology Before Pregnancy and Female Hormones
Chapter 83. Pregnancy and Lactation
Chapter 84. Fetal and Neonatal Physiology
Unit XV. Sports Physiology
Chapter 85. Sports Physiology
Chapter 1. Functional Organization of the Human Body and Control
of the
“Internal Environment”
Test Bank
1. The most abundant type of cell in the human body is which of the following?
A. Neuron
B. Epithelial cell
C. Red blood cell
D. White blood cell
E. Vascular smooth muscle cell
F. Skeletal muscle cell
ANSWER: C
2. The most abundant substance in the human body and the approximate
percentage ofthat substance in the body is which of the following?
A. Protein, 30%
B. Protein, 60%
C. Water, 30%
D. Water, 60%
E. Carbohydrate, 30%
F. Carbohydrate, 60%
ANSWER: D
3. A large volume of blood is transfused to a person whose baroreceptor blood
pressure control system is not functioning and arterial blood pressure rises from
the normal level of 100 to 160 mm Hg. If the same volume of blood is infused
into the same person when the baroreceptor system is functioning and this time
the arterial pressure increases from the normal level from 100 mm Hg up to 120
mm Hg, calculate the gain of the baroreceptor system in this person.
A. -3
B. -2
C. -1
D. 0
E. +1
F. +2
G. +3
ANSWER: B
4. Which of the following substances has the highest extracellular fluid to
intracellular fluid concentration ratio for most mammalian cells?
A. Sodium ions
B. Potassium ions
C. Carbon dioxide
D. Glucose
E. Protein
ANSWER: A
5. Exchange of substances between the cardiovascular system and the
interstitial fluid occurs mainly in which of the following?
A. Arteries
B. Arterioles
C. Capillaries
D. Venules
E. Veins
ANSWER:
C
6. Which of the following is the approximate distance from the capillaries to most
cellsof the body?
A. Less than 50 angstroms
B. Less than 50 microns
C. Less than 50 millimeters
D. Less than 100 angstroms
E. Less than 100 microns
F. Less than 100 millimeters
ANSWER: A
7. When a person is at rest, how much time is required for the blood in the
circulation to traverse the entire circulatory circuit?
A. 1 second
B. 1 minute
C. 3 minutes
D. 4 minutes
E. 5 minutes
ANSWER: B
8. feedback is often referred to as a "vicious cycle" because it leads to
instability and sometimes death.
A. Postitive, progressive
B. Positive, diminished
C. Negative, progressive
D. Negative, diminished
E. Adaptive, progressive
ANSWER: A
9. Which of the following is an example of positive feedback in the body?
A. Clotting of blood
B. Return of blood pressure toward normal after a hemorrhage
C. Increased respiration rate caused by accumulation of carbon dioxide in the
blood
D. Decreased sympathetic nervous system activity that occurs in response to
increased blood pressure
ANSWER: A
Chapter 2. The Cell and Its Functions
Test Bank
Refer to the following list to answer questions 1-3:
A. Nucleolus
B. Nucleus
C. Agranular endoplasmic reticulum
D. Granular endoplasmic reticulum
E. Golgi apparatus
F. Endosomes
G. Peroxisomes
H. Lysosomes
I. Cytosol
Identify the cellular location for each of the following steps involved in the
synthesis and packaging of a secreted protein.
Initiation of translation.
ANSWER: I
Protein sorting and packaging.
ANSWER: E
Gene transcription.
ANSWER: B
4. Which of the following is true for both pinocytosis and phagocytosis?
A. Involves the recruitment of actin filaments
B. Occurs spontaneously and non-selectively
C. Permits the uptake of bacterium into the cytosol
D. Is only observed in macrophages and neutrophils
E. Does not require ATP
ANSWER: A
5. The cell membrane is LEAST permeable to which of the following?
A. Sodium
B. Oxygen
C. Ethanol
D. Carbon Dioxide
E. Water
ANSWER: A
6. The term “glycocalyx” refers to:
A. The negatively charged carbohydrate chains that protrude into the
cytosol from glycolipids and integral glycoproteins
B. The negatively charged carbohydrate layer on the outer cell surface
C. The layer of anions aligned on the cytosolic surface of the plasma membrane
D. The large glycogen stores found in “fast” muscles
E. A mechanism of cell-cell attachment
ANSWER: B
7. Proteins are sorted for their delivery to lysosomes, secretory vesicles and the
plasma membrane in the:
A. Golgi apparatus
B. smooth endoplasmic reticulum
C. nucleus
D. endocytotic vesicle
ANSWER: A
8. Ubiquinone, an electron acceptor in the electron transport chain
(oxidative phosphorylation), is found in the:
A. Inner mitochondrial membrane
B. Mitochondrial matrix
C. Outer mitochondrial membrane
D. Nucleus
ANSWER: A
9. The citric acid cycle or Kreb’s cycle, takes place in the:
A. Mitochondrial matrix
B. Inner mitochondrial membrane
C. Outer mitochondrial membrane
D. Inner mitochondrial space
ANSWER: A
10. Which of the following processes is NOT ATP-dependent?
A. Ciliary movement
B. Positive chemotaxis
C. Movement of carbon dioxide across a lipid bilayer
D. Endocytosis
E. Smooth muscle contraction
ANSWER: C
11. This cytoskeletal element plays a role in certain forms of cell movement and
is an essential component of the mitotic spindle:
A. Phospholipids
B. Glycocalyx
C. F-actin
D. Microtubules
E. Clathrin
ANSWER: D
12. Lipid synthesis occurs in the:
A. Trans-Golgi network
B. Granular or “rough” endoplasmic reticulum
C. Agranular or “smooth” endoplasmic reticulum
D. Nucleus
E. Lysosome
ANSWER: C
13. This cytoskeletal element plays a role in certain forms of cell movement
and is an essential component of the mitotic spindle:
A. Phospholipids
B. Glycocalyx
C. F-actin
D. Microtubules
E. Clathrin
ANSWER: D
14. The abnormal cleavage of mannose residues during the post-translational
processing of glycoproteins has been shown to result in the development of a
lupus-like autoimmune disease in mice. The abnormal cleavage is due to a
mutation of the enzyme -mannosidase II.
Based on your understanding of the processing of membrane proteins, you
would predict this enzyme to be localized to the:
A. Nucleus
B. Cytosol
C. Golgi apparatus
D. Lysosomes
E. Peroxisomes
ANSWER: C
15. The observation that abnormal cleavage of mannose residues from
glycoproteins causes an autoimmune disease in mice is most consistent with
the role of which of the following structures in the normal immune response?
A. Cytoskeleton
B. Glycocalyx
C. Peroxisomes
D. Lysosomes
E. Microtubules
ANSWER: B
16. A pure phospholipid bilayer is most permeable to:
A. Sodium
B. Calcium
C. Chloride
D. Water
E. Oxygen
ANSWER: E
Chapter 3. Genetic Control of Protein Synthesis, cell function, and
cell reproduction
Test Bank
1. Facioscapulohumeral muscular dystrophy (FSHD) is characterized by the
deletion of a DNA sequence on chromosome 4q35. This deletion correlates with
both the inability of a specific protein complex to bind to the DNA and an
overexpression of the genes upstream of the deletion. The sequence deleted in
FSHD most likely functions normally as:
A. An activator protein
B. A repressor protein
C. An activator element
D. A repressor element
E. A promoter sequence
ANSWER: D
2. In comparing two cell types from the same person (e.g. a neuron and an
epithelial cell) the variation in their proteomes, or the proteins expressed by
each cell type, reflects:
A. Differences in the DNA contained in the nucleus of each cell
B. Variation in the numbers of copies of specific genes in their respective genomes
C. Cell-dependent expression and/or repression of specific genes
D. Differences in the number of chromosomes in each cell
E. The loss of genes from each genome over time
ANSWER: C
3. Which of the following correctly describes the sequence of events that occur
during the synthesis and packaging of a secreted protein?
A. The gene is transcribed in the cytosol; mRNA is translated by ribosomes
bound to “rough” endoplasmic reticulum; the protein is packaged for
secretion in the trans- Golgi network
B. The gene is transcribed in the nucleus; RNA polymerase binds to the start
codon; the protein is packaged for secretion in the trans-Golgi network
C. Translation is initiated in the cytosol; the protein is carried in membrane-
bound vesicles to the cis-Golgi apparatus; the protein is packaged for
secretion in the trans- Golgi network
D. Translation is initiated in the cytosol; new polypeptide is co-transported
with Na
+
across the outer ER membrane; the protein is glycosylated in
the Golgi apparatus
E. Translation is initiated by small ribosomal subunits anchored to the ER
membrane; protein is glycosylated in the Golgi apparatus; the protein is
packaged into lysosomes
ANSWER: C
4. Which of the following does NOT play a direct role in the process of transcription?
A. Helicase
B. RNA polymerase
C. A chain terminating sequence
D. “Activated” RNA molecules
E. A promoter sequence
ANSWER: A
5. “Redundancy” or “degeneration” of the genetic code occurs during which of the
following steps of protein synthesis?
A. DNA replication
B. Transcription
C. Post-transcriptional modification
D. Translation
E. Protein glycosylation
ANSWER: B
6. Which of the following bases is NOT present in RNA?
A. Cytosine
B. Thymine
C. Adenine
D. Guanine
ANSWER: B
7. The process of translation takes place:
A. In the cytosol and on the surface of the “rough” endoplasmic reticulum
B. In the nucleus and on the surface of the “rough” endoplasmic reticulum
C. In the cytosol and the trans-Golgi network (TGN)
D. In the nucleus and on the outer mitochondrial membrane
ANSWER: A
8. Which of the following statements about translation is NOT true?
A. Multiple ribosomes can simultaneously translate a single mRNA molecule
B. Each codon codes for one amino acid
C. One mRNA molecule can code for multiple proteins, depending on which start
codon is recognized by the small ribosomal subunit
D. Translation is terminated when the release factor binds to the stop codon
ANSWER: C
9. The following statements accurately describe the process of DNA
replication EXCEPT:
A. The entire genome is replicated only once per cell cycle
B. It occurs during the M phase of the cell cycle
C. Nucleotides are incorporated into the growing DNA strand at the 3’ end
D. DNA “proofreading” is performed by DNA polymerase
ANSWER: C
10. The mechanism by which allolactose regulates the transcription of the
beta- galactosidase gene is best described as:
A. De-repression
B. Repression
C. Activation
D. Negative feedback
E. Positive feedback
ANSWER: A
11. Which of the following does NOT occur during the process of mitosis?
A. Replication of the genome
B. Condensation of the chromosomes
C. Fragmentation of the nuclear envelope
D. Alignment of the chromatids along the equatorial plate
E. Separation of the chromatids into two sets of 46 “daughter” chromosomes
ANSWER: A
12. “Redundancy” or “degeneration” of the genetic code occurs during which of the
following steps of protein synthesis:
A. DNA replication
B. Transcription
C. Post-transcriptional modification
D. Translation
E. Protein glycosylation
ANSWER: B
13. The appearance of which of the following distinguishes eukaryotic cells from
lower units of life like bacteria and viruses?
A. DNA
B. RNA
C. Membranes
D. Protein
E. Nucleus
ANSWER: E
Chapter 4. Transport of Substances Through Cell Membranes
Test Bank
1. An artificial membrane is created consisting of a lipid bilayer without protein
molecules in the membrane. The lipid composition of the membrane is
essentially the same as that of a normal, biological membrane. Which of the
following substances permeates the membrane more readily than water
molecules?
A. Carbon Dioxide
B. Glucose
C. Glycerol
D. Sodium
E. Urea
ANSWER:
A
2. A cell is equilibrated in an aqueous solution of 300 mOsm/L sodium chloride.
Which of the following best describes what will happen to cell volume when the
cell is placed in an aqueous solution of 300 mOsm/L calcium chloride?
A. Decrease
B. Decrease and then increase
C. Increase
D. Increase and then decrease
E. No change
ANSWER: E
3. The intracellular calcium ion concentration of ventricular muscle cells averages
10
-4
mmol/L during diastole. The calcium ion concentration in transverse
tubules (T-tubules) averages 2.5 mmol/L at rest. A protein transporter on the
membrane of the T-tubule exchanges sodium for calcium. The transporter uses
the transmembrane sodium gradient to fuel the exchange. Which of the
following transport mechanisms best describes this type of transporter?
A. Facilitated diffusion
B. Primary active transport
C. Secondary active co-transport
D. Secondary active counter-transport
E. Simple diffusion
ANSWER: D
4. Human red blood cells (RBCs) and rabbit RBCs are equilibrated in separate
solutions of isotonic saline (300 mOsm/L NaCl). The human RBCs are then
placed in a solution of 300 mOsm/L glycerol, which causes them to swell and
burst. However, rabbit RBCs placed in 300 mOsm/L glycerol neither swell nor
shrink. Based on this information, which of the following can be concluded
about a 300 mOsm/L solution of glycerol for the different cell types?
Human RBCs Rabbit RBCs
A. Hypertonic and hyperosmotic Hypotonic and hypoosmotic
B. Hypotonic and hypoosmotic Hypertonic and hyperosmotic
C. Hypotonic and isoosmotic Isotonic and isoosmotic
D. Isotonic and hypoosmotic Isotonic and hyperosmotic
E. Isotonic and isoosmotic Hypotonic and isoosmotic
F. Isotonic and hyperosmotic Isotonic and isoosmotic
ANSWER: C
5. The molarity of a 2% solution of NaCl is 340 mmol/L. The molecular weight of NaCl
is
58.5. What is the osmolarity of a 2% solution of NaCl (in
mOsm/L)? A. 170
B. 340
C. 510
D. 680
ANSWER
: D
6. Secondary active transport typically moves which of the following substances
against a concentration gradient?
Glucose
A. No
Amino acids
No
Sodium ions
No
B. No No Yes
C. Yes No Yes
D. Yes
E. Yes
Yes
Yes
No
Yes
ANSWER: D
7. Which of the following transport mechanisms can move sodium ions across
a cell membrane?
Primary active
transport
No
No
No Yes
Secondary
active
transport
No
Yes
Simple
diffusio
n
Yes No Yes
Yes Yes No
Yes Yes Yes
ANSWER: E
8. The diagram illustrates possible changes in red blood cell volume resulting
from a change in extracellular fluid composition for a cell equilibrated in a 150
mmol/L solution of sodium chloride (NaCl) at time zero. Which curve best
illustrates the volume change caused by immersion of the cell in an aqueous
solution of 300 mOsm/L calcium chloride (CaCl2)?
ANSWER: C
9. The diagram illustrates possible changes in red blood cell volume resulting
from a change in extracellular fluid composition for a cell equilibrated in a 150
mmol/L solution of sodium chloride (NaCl) at time zero. Which curve best
illustrates the volume change caused by immersion of the cell in an aqueous
solution of 200 mOsm/L NaCl and 200 mOsm/L glycerol?
ANSWER: B
10. Which of the following pairs of aqueous solutions will exert equal osmotic
pressures across a normal cell membrane after steady-state conditions have
been established?
A.
Solution A
10% albumin
Solution B
10% IgG
B.
C.
100 mmol/L NaCl
300 mOsm/L glucose
200 mmol/L
CaCl2
300 mOsm/L urea
D.
E.
300 mOsm/L glycerol
300 mOsm/L glycerol
300 mOsm/L
NaCl
300 mOsm/L urea
ANSWER: E
11. Two compartments (X and Y) are separated by a typical biological membrane
(i.e., lipid bilayer). The concentrations of a permeant solute (i.e., urea) at time
zero are shown. Which of the drawings below represents the volumes of X and Y
when the system reaches equilibrium?
ANSWER: A
12. The diagram illustrates possible changes in red blood cell volume resulting
from a change in extracellular fluid composition for a cell equilibrated in 150
mmol/L NaCl at
time zero. Which curve best illustrates the volume caused by immersion of the
cell in an aqueous solution of 150 mmol/L CaCl2?
ANSWER: E
13. Two compartments (X and Y) are separated by a typical biological membrane
(lipid bilayer). The concentrations of a non-permeant molecule (glucose) at
time zero are shown. Which of the drawings below represents the volumes of X
and Y when the system reaches equilibrium?
ANSWER: B
14. The diagram shows a model cell that transports substance X across the cell
membrane. The cell is equipped with a Na-K-ATPase pump as shown. Substance
X enters the cell by a coupled transport mechanism and exits the cell by
carrier-mediated diffusion. Treatment with a substance that inhibits the Na-K-
ATPase pump inhibits the transport of X by which of the following mechanisms?
A. Decreasing intracellular K
+
concentration
+
B. Decreasing intracellular
Na
+
concentration
C . Increasing intracellular K concentration
D. Increasing intracellular Na
+
concentration
ANSWER: D
15. The diagram shows a bag (with permeability characteristics similar to that of a
normal cell) that contains a 100 mM solution of urea at time zero. The bag is
placed in a beaker containing 100 mM glucose. Which of the following best
describes the tonicity and osmolarity of the glucose solution as well as any
changes in bag volume (assume that the bag volume is infinitely small
compared to beaker volume)?
Osmolarity Tonicity Bag volume
A. Hyperosmotic Hypertonic Decreases
B. Hyperosmotic Hypotonic Increases
C. Hyperosmotic Isotonic No change
D. Hypoosmotic Hypotonic Decreases
E. Hypoosmotic Isotonic Increases
F. Hypoosmotic Hypertonic No change
G. Isoosmotic Hypertonic Decreases
H. Isoosmotic Hypotonic Increases
I. Isoosmotic Isotonic No change
ANSWER: G
Chapter 5. Membrane Potentials and Action Potentials
Test Bank
1. The diagram shows compound action potentials recorded percutaneously
from a single site a certain distance from a stimulating electrode indicated by
the arrow. Which peak is most likely to represent action potentials from small,
non-myelinated motor neurons?
ANSWER: D
2. A 42-year-old woman decides to lose weight on a diet prescribed by an
anorexic friend. She loses about 30 pounds in 45 days, but her serum
potassium level falls to 2.1 mmol/L (normal: 3.5 to 5.0 mmol/L). Which of the
following changes is most likely to occur in this young woman?
A. Depolarization of the resting membrane potential
B. Hyperpolarization of the resting membrane potential
C. Increased intracellular potassium concentration
D. Potassium equilibrium potential becomes less negative
ANSWER: B
3. An 22-year-old man is admitted to the emergency department after an
automobile accident. He has not lost a large amount of blood, but he suffers
from a severe crush injury to his right leg. He is conscious and exhibits flaccid
paralysis. Electrocardiogram
Na
results show a shortened QT interval and an increase in the P-R interval (i.e.,
decreased duration of action potential and decreased conduction velocity).
Laboratory results indicate that his serum potassium level has increased to a
value of 6.5 mEq/L (normal:
3.5 to 5.0 mEq/L). This increase in serum potassium causes which of the following?
A. A decreased net driving force for potassium from cells
B. Decreased intracellular potassium concentration
C. Hyperpolarization of the resting membrane potential
D. More negative Nernst potential for potassium
E. More negative resting membrane potential
ANSWER: A
4. An 88-year-old nursing home resident has diabetes mellitus and is taking
diuretics for hypertension. His blood glucose concentration has recently been
greater than 350 mg/dL. He is disoriented and has had limited oral intake for
about five days. His vital signs include a temperature of 100.5°F, a supine
blood pressure and pulse of 150/90 mm Hg and 100 beats per minute, and an
upright blood pressure and pulse of 130/88 mm Hg and 118 beats per minute. The
findings on examination of the heart are normal, and his axillae
+ +
a re dry. The table shows serum N
K
a and values before and after the
dehydration that has occurred in this patient. The
resting membrane potential of a cardiac myocyte was about −70 millivolts before
dehydration occurred. What is the resting membrane potentialof a cardiac
myocyte after dehydration occurred, assuming no change in membrane
conductance or intracellular concentrations of
+
or K
+
ions?
A. −100 millivolts
B. −50 millivolts
C. −60 millivolts
D. −70 millivolts
E. −90 millivolts
ANSWER: D
5. A 44-year-old man is recovering from third degree burns over 50% of his body
surface area. His serum potassium level is low (2.9 mEq/L). He exhibits postural
hypotension and skeletal muscle weakness. If the equilibrium potential for K
+
in a neuron is −105 mV and the equilibrium potential for Na
+
is +55 mV, what
would be an approximate value for the resting membrane potential for a
neuron in this man?
A. +20 mV
B. +48 mV
C. +80 mV
D. −25 mV
E. −48 mV
F. −96 mV
ANSWER: F
6. A 23-year-old man receives a crush injury to his right leg in an automobile
accident. His serum potassium level is 6.4 mEq/L. The equilibrium potential for
K
+
in a motoneuron is −73 mV, and the equilibrium potential for Na
+
is +59
mV. Which of the following best describes the resting membrane potential (in
mV) for a typical motoneuron in this man?
A. +62
B. +90
C. −13
D. −67
E. −84
ANSWER: D
7. A 56-year-old woman is referred to the neurology clinic for evaluation of her
long- term problem of a stumbling gait and a tendency to fall. Her visual acuity
also seems to change periodically. Neurological findings suggest multiple
sclerosis. This diagnosis is further supported by magnetic resonance imaging,
which shows of areas of demyelination in the central nervous system. Other
tests show the presence of oligoclonal bands in the spinal fluid. The loss of myelin
from a nerve fiber has which of the following effects?
A. Decreased conduction velocity of action potential or no action potential
B. Decreased energy requirement for successful action potentials
C. Increased conduction velocity of action potential
D. Increased velocity of salutatory conduction
ANSWER: A
8. The diagram shows an action potential for a nerve fiber. What is the threshold
potential for this neuron?
A. −15 mV
B. −50 to −60 mV
C. −60 to −70 mV
D. −70 to −80 mV
E. −80 mV
ANSWER: C
9. In the diagram shown, Em represents the measured initial membrane potential
for a hypothetical cell in vivo. In relation to this membrane potential, the
equilibrium potentials
of three ions (X, Y, Z) are represented. Pick the path most likely taken by the
membrane potential when membrane conductance for ion Y is increased.
ANSWER: B
10. In the diagram shown, Pmemb represents the initial membrane potential for a
hypothetical cell. In relation to this membrane potential, the equilibrium
potentials of three ions (Q, R, S) are represented. Pick the path most likely
taken by the membrane potential when membrane conductance for ion Q and
ion S are increased simultaneously by 1000-fold.
ANSWER: C
11. The diagram shows compound action potentials recorded percutaneously from a
single site at a certain distance from a stimulating electrode (S) indicated by the
ARROW. What can be concluded from the diagram concerning action potential
velocity for the various categories of neurons?
A. W = X = Y = Z
B. W > X > Y > Z
C. W > Y > X
D. Y > W > Z
E. Z > Y > X > W
ANSWER: B
12. Five hypothetical neurons are shown below. Neurons A and B are myelinated,
whereas neurons C, D, and E are not myelinated. Which neuron is most likely
to have the fastest conduction velocity?
A.
B.
C.
D.
E.
ANSWER: B
13. In the following graph, Em represents the measured initial membrane potential
for a hypothetical cell in vivo. In relation to this membrane potential, the
equilibrium potentials of three ions (Q, R, S) are represented. During resting
conditions, the conductance for Q and S ions is the same, which is 1000 times
greater than that of ion R. Pick the path most likely taken by the membrane
potential when membrane conductance to ion Q is
2
decreased to 0 mmho/cm .
ANSWER: D
14. A 63-year-old man is dehydrated after three days of alcohol abuse. His serum
sodium level has increased from a normal value of 140 mEq/L to 147 mEq/L.
The resting membrane potential of a typical neuron was −80 mV before
dehydration. What is the most likely resting membrane potential of a
typical neuron after the dehydration, assuming no change in resting sodium
conductance?
A. −72 mV
B. −74 mV
C. −80 mV
D. −88 mV
E. −98 mV
ANSWER: C
15. A 24-year-old soldier suffers a burn injury over 60% of his body surface area, which
+
l eads to a decrease in the serum K concentration from a normal value of 4
mEq/L to a value of 2.5 mEq/L. Which of the
following sets of changes best describe the resting membrane potential and
the K
+
Nernst potential in a cardiac myocyte after the injury?
+
Resting membrane potential K Nernst potential
A. Less negative Less negative
B. Less negative More negative
C. More negative Less negative
D. More negative More negative
E. No change No change
ANSWER: D
Ca
16. The diagram shows an action potential for a nerve fiber. Use the diagram to
answer the question. Where on the curve does potassium conductance exceed
sodium conductance?
A. V and Z
B. V, W, and X
C. V, X, and Z
D. W, X, and Y
E. X, Y, and Z
ANSWER: A
17. Consider the action potential shown. Which of the following changes in membrane
conductances for
+ and
K
+
best account for the prolonged plateau (assume normal
conductances for both ions during resting conditions, i.e., only the changes in
conductances are important)?
A.
B.
C.
D.
ANSWER: A
18. A neuron from an extraterrestrial being has the intracellular and extracellular
sodium and potassium concentrations shown in the table (in mM). During
resting conditions, the sodium conductance is 100-fold greater compared to the
potassium conductance. Which of the following (in mV) best describes the
resting membrane potential (Vm) in this alien neuron (assume that Vm is
dictated mainly by sodium and potassium conductances and that the Nernst
equation applies)?
A. +150
B. +55
C. 0
D. −55
E. −150
ANSWER:
B
19. The intracellular concentration of ion Z is 100 milliequivalents per liter (mEq/L),
and the extracellular concentration is 5 mEq/L. The molecular weight of
molecule Z is ~40 daltons. This relatively high intracellular concentration of
molecule Z is best explained by which of the following?
A. Exocytosis
B. Extracellular assembly
C. Facilitative diffusion
D. Intracellular production
E. Primary active transport
F. Simple diffusion
ANSWER: E
20. Equilibrium potentials for Na, K, and Cl are shown. Assume that a cell is
permeable to all three ions and has a resting membrane potential of −85 mV.
Which of the following best describes the net movement of the various ions across
the cell membrane by way of passive diffusion?
Sodium Potassium Chloride
A. Inward Inward Inward
B. Inward Inward Outwar
d
C. Inward Outward Inward
D. Inward Outward Outwar
d
E. Outward Inward Inward
F. Outward Inward Outwar
d
G. Outward
H. Outward
Outward
Outward
Inward
Outwar
d
ANSWER: D
Chapter 6. Contraction of Skeletal Muscle
Test Bank
1. A 28-year-old man has been training as a distance runner for the past six years.
Compared with a sprinter, which of the following is decreased in the skeletal
muscles of this athlete during resting conditions?
A. Capillary density
B. Glycolytic enzyme content
C. Mitochondrial volume density
D. Myoglobin content
E. Oxidative capacity
ANSWER: A
2. A physiology experiment is conducted in which a skeletal muscle twitch is
initiated using an electrical stimulator. Which factor contributes to the
termination of a typical skeletal muscle twitch?
A. Calcium uptake by sarcoplasmic reticulum
B. Closure of transverse tubules
C. Depletion of calcium stores from sarcoplasmic reticulum
D. Exhaustion of muscle ATP stores
E. Stimulation of dihydropyridine (DHP) receptors
ANSWER: A
3. A 24-year-old medical student goes to the local gym to lift weights. She begins
by bench- pressing 100 pounds as a warm-up procedure and then gradually
increases the weight. Which of the following occurs as she adds more weight?
A. Decreased frequency of motor nerve action potentials
B. Decreased velocity of motor nerve action potentials
C. Increased frequency of motor nerve action potentials
D. Increased velocity of motor nerve action potential
E. Involvement of fewer motor units
ANSWER: C
4. A 24-year-old woman is admitted to the emergency department after an
automobile accident in which lacerations to the left wrist severed a major
tendon. The severed ends of the tendon were overlapped by 6 cm to facilitate
suturing and reattachment. Which of the following characteristics would be
expected after six weeks compared with the pre- injured muscle? Assume that
series growth of sarcomeres cannot be completed within six weeks.
Passive tension Maximal active tension
A. Decrease Decrease
B. Decrease Increase
C. Increase Increase
D. Increase Decrease
E. No change No change
ANSWER: D
5. During the contraction of a skeletal muscle fiber, the actin and myosin
filaments slide past each other. Which of the following represent expected
changes in the widths of I- bands and A-bands during the contraction process?
I-band width A-band
width
A.
B.
C.
D.
E.
ANSWER: D
6. When a person lifts a 25-pound weight, what type or types of muscle
contraction are involved?
Isometric contraction only
Isotonic contraction only
Isometric contraction followed by isotonic contraction
Isotonic contraction followed by isometric contraction
ANSWER: C
7. The length-tension diagram shown was obtained from a skeletal muscle with
equal numbers of red and white fibers. Supramaximal tetanic stimuli were used
to initiate an isometric contraction at each muscle length studied. The resting
length was 20 cm. What is the maximum amount of active tension this muscle
is capable of generating at a preload of 100 grams?
A. 145 to 155 grams
B. 25 to 35 grams
C. 55 to 65 grams
D. 95 to105 grams
E. Cannot be determined
ANSWER: C
8. The diagram shows the relationship between muscle tension and sarcomere
length for muscle. Which point on the curve represents tension development
at a normal resting length for skeletal muscle?
ANSWER: C
9. The diagram shows the relationship between muscle tension and sarcomere
length for muscle. Which point on the curve represents tension development
at a normal resting length for cardiac muscle?
ANSWER: B
10. A 29-year-old woman has been training as a distance runner for the past five
years. Which set of changes best describes the skeletal muscles of this woman
compared with a sprinter?
Capillary density Oxidative capacity Myoglobin content Fiber diameter
A.
B.
Decrease
d
Decrease
d
Decreased
Decreased
Decreased
Increased
Decrease
d
Increased
C. Increased Increased Decreased Decrease
d
D.
E.
Increased
Increased
Increased
Increased
Increased
Increased
Decrease
d
Increased
ANSWER: D
11. The length-tension diagram shown on the right was obtained from a skeletal
muscle with equal numbers of red and white fibers. Supramaximal tetanic
stimuli were used to initiate an isometric contraction at each muscle length
studied. What are the values in grams of preload, active tension, and total
tension when the muscle length is 60 cm?
A.
Preloa
d 100
Active
tension 200
Total
tension 300
B. 100 300 200
C. 150 250 300
D. 200 100 300
E. 200 200 300
ANSWER: A
12. A 45-year-old man goes to the local gym to lift weights. He begins by bench-
pressing 130 pounds as a warm-up procedure and then gradually increases
the weight. Which of the following sets of changes occur as he adds more
weight?
A.
Activation of motor units
Decreased
Frequency of motor nerve action
potentials
Decreased
B.
C.
Decreased
Decreased
Increased
No change
D.
E.
F.
Increased
Increase
d
Increase
d
Decreased
Increased
No
change
ANSWER: E
13. The skeletal muscle sarcomere normally has a narrow range of lengths in vivo,
but sarcomere length can vary greatly in vitro under non-physiological
conditions. The diagram shows sarcomeres at various lengths. At which length
can the sarcomere exert its maximum force during an isometric contraction?
A. 1.6 µm
B. 2.1 µm
C. 2.8 µm
D. 3.0 µm
E. 3.5 µm
ANSWER: B
14. The diagram shows the relationship between contraction velocity and force.
Which of the following muscles is most likely to correspond to muscle number 1
shown on the diagram (assume that all muscles shown are at their normal
resting lengths).
A.
B.
C.
D.
E.
ANSW
ER: E
15. The diagram shows the relationship between contraction velocity and force.
Which of the following muscles is most likely to correspond to muscle number 2
shown on the diagram (assume that all muscles shown are at their normal
resting lengths)?
A.
B.
C.
D.
E.
16. The diagram shows force-velocity relationships for three different skeletal
muscles. Which of the following curves most likely correspond to glycolytic,
oxidative, and mixed muscle types?
Glycolytic Oxidative Mixed
A. X Y Z
B. X Z Y
C. Y X Z
D. Y Z X
E. Z
F. Z
X
Y
Y
X
ANSWER: B
17. The dark color of red skeletal muscle (slow twitch muscle) can be attributed to a
relative abundance of which of the following compared to white skeletal
muscle (fast twitch muscle)?
A. Blood capillaries
B. Mitochondria
C. Myoglobin
D. Myosin ATPase
E. Oxidative enzymes
ANSWER: C
Chapter 7. Excitation of Skeletal Muscle: Neuromuscular Transmission
and
ANSW
ER: D
Excitation-Contraction Coupling
Test Bank
1. A 45-year-old man has botulinum toxin (Botox®) injections in the frontalis
muscles. Which of the following sets of changes are most likely to occur in the
treated muscles with repeated injections over a period of ten years?
Fiber diameter Fiber number Contraction
velocity
A. decreased decreased decreased
B. decreased decreased increased
C. decreased increased increased
D. increased decreased decreased
E. increased
F. increased
decreased
increased
increased
increased
ANSWER: A
2. Which of the following represents the correct temporal order of events for
skeletal muscle?
First
A. End plate potential
Second
Muscle action potential
Third
Nerve action
potential
B. End plate potential Nerve action potential Muscle action
potential
C. Muscle action potential End plate potential Nerve action
potential
D. Muscle action potential Nerve action potential End plate potential
E. Nerve action potential
F. Nerve action potential
End plate potential
Muscle action potential
Muscle action
potential
End plate potential
ANSWER: E
3. Myasthenia gravis is an autoimmune disease in which antibodies damage
which of the following?
Acetylcholine
Ligand-gated channels on postsynaptic
membrane Ligand-gated channels on
presynaptic membrane Voltage-gated
channels on postsynaptic membrane Voltage-
gated channels on presynaptic membrane
ANSWER: B
4. Lambert-Eaton myasthenic syndrome is an autoimmune disease in which
antibodies damage which of the following?
A. Acetylcholine
B. Ligand-gated channels on postsynaptic membrane
C. Ligand-gated channels on presynaptic membrane
D. Voltage-gated channels on postsynaptic membrane
E. Voltage-gated channels on presynaptic membrane
ANSWER: E
5. A 59-year-old college professor visits his physician because of muscle fatigue
that worsens later in the day. The physician notices that the man has droopy
eyelids and seems to have an overall weakness in his muscles. After the
intravenous administration of Tensilon®, which blocks degradation of
acetylcholine, the droopy eyelids and symptoms of muscle weakness appear to
subside. Which of the following best describes the patient’s condition?
A. Encephalitis
B. Fibromyalgia
C. Flaccid paralysis
D. Myasthenia gravis
E. Poliomyelitis
ANSWER: D
6. A 3-year-old child is admitted to the emergency department of University
Hospital. The child exhibits extreme salivation, lacrimation, tremors, and
tachycardia. The slight erythema and mild edema found on the child's hand
suggest a spider bite. The mother acknowledges that the child has a
fascination with spiders and has seen black widow spiders in the yard. The
neurotoxic venom of the black widow spider (i.e., latrotoxin or latrophilin) can
increase the flux of calcium ions into the presynaptic terminal. What is the
likely outcome of a black widow spider bite?
A. Decreased acetylcholine release from nerve terminals
B. Hyperpolarization of postsynaptic membranes
C. Hyperpolarization of presynaptic membranes
D. Increased acetylcholine release from nerve terminals
E. Inhibition of acetylcholinesterase
ANSWER: D
7. An apparently healthy 12-year-old boy dies during a minor surgical procedure
while under general anesthesia. An uncle had also died during a surgical
procedure. Results of a halothane contracture test indicate that the child had
malignant hyperthermia (MH). The MH crisis is associated with which of the
following?
A. Decreased anaerobic metabolism
B. Decreased CO2 production by muscles
C. Decreased lactic acid production by muscles
D. Defective ryanodine receptors
E. Decreased usage of ATP in muscles
ANSWER: D
8. Skeletal muscle and cardiac muscle contract when calcium binds to which
structure or molecule?
Active sites
Calmodulin
Myosin
head
Tropomyosi
n Troponin-
C
ANSWER: E
9. Which of the following represents the correct temporal order of events for
skeletal muscle?
First Second Third
Muscle action potential Muscle contraction Nerve action
potential Muscle action potential Nerve action
potential Muscle contraction
Muscle contraction Muscle action potential Nerve action
potential Muscle contraction Nerve action potential Muscle action
potential Nerve action potential Muscle action
potential Muscle contraction Nerve action potential
Muscle contraction Muscle action
potential
ANSWER: E
10. Binding of calcium to which structure or molecule initiates a contraction in
smooth muscle, skeletal muscle, and cardiac muscle?
Smooth muscle Skeletal muscle Cardiac
muscle
A. Active sites Calmodulin Troponin
B. Active sites Troponin Calmodulin
C. Calmodulin Tropomyosin Active sites
D. Calmodulin
E. Myosin heads
F. Myosin heads
Troponin
Tropomyosi
n Troponin
Troponin
Troponin
Tropomyosi
n
ANSWER: D
11. A 23-year-old man loses consciousness while driving, hits a tree, and dies
within 15 minutes. The police find his stiff body five hours later. Rigor mortis
is caused by a decrease in which of the following?
A. Acetylcholine
B. Actin-myosin cross-bridges
C. Myoplasmic calcium levels
D. Interstitial lactate levels
E. Muscle ATP levels
ANSWER: E
12. A father and his 10-year-old daughter died suddenly of suffocation after
ingesting smoked whitefish chubs from the Great Lakes. Three other family
members are sick,
complaining of dry mouths despite drinking copious amounts of fluids. Their
upper eyelids droop. Their vision is clear, but their pupils are wide and do not
narrow when a light is flashed. Two of them require artificial respiration. The
suspect is botulism (Clostridium botulinum). The surviving patients begin
improving soon after receiving the only treatment available, E antitoxin. The
deadly effect of botulinum toxin results from which of the following?
A. Decreased release of acetylcholine
B. Decreased release of norepinephrine
C. Depletion of muscle calcium stores
D. Skeletal muscle tetany
E. Stimulation of ryanodine receptors
ANSWER: A
13. Which of the following best describes the dynamics of calcium release and
calcium uptake during the development of frequency-induced muscle
tetany?
A. Release from longitudinal tubules exceeds uptake by terminal cisternae
B. Release from longitudinal tubules exceeds uptake by T-tubules
C. Release from terminal cisternae exceeds uptake by longitudinal tubules
D. Release from terminal cisternae exceeds uptake by T-tubules
E. Release from T-tubules exceeds uptake by longitudinal tubules
F. Release from T-tubules exceeds uptake by terminal cisternae
ANSWER: C
14. Which of the following best describes the correct temporal order of events for
the opening of nicotinic acetylcholine (nACh) channels on the muscle
membrane, the opening of voltage-gated sodium channels on the muscle
membrane, and initiation of presynaptic action potentials (AP)?
First
A. nACh channels
Second
Presynaptic AP
Third
Voltage gated Na
channels
B. nACh channels Voltage gated Na
channels
Presynaptic AP
C. Presynaptic AP nACh channels Voltage gated Na
channels
D. Presynaptic AP Voltage gated Na
channels
nACh channels
E. Voltage gated Na
channels
F. Voltage gated Na
channels
nACh channels
nACh channels
Presynaptic AP
Presynaptic AP
ANSWER: C
Chapter 8. Excitation and Contraction of Smooth Muscle
Test Bank
1. The contraction of smooth muscle can be attenuated even when intracellular
calcium levels are high. This unique ability of smooth muscle can be attributed to
variations in the activity of which of the following?
A. Actin
B. Adenosine triphosphate (ATP)
C. Calcium-calmodulin complex
D. Calmodulin
E. Myosin light chain phosphatase
ANSWER: E
2. Multiple elements can regulate the contraction of smooth muscle, including
endocrine, neurocrine, and paracrine factors. This level of diversity in the
control of smooth muscle contraction can be attributed to a diversity of which
of the following?
A. Actin-myosin arrangement
B. Calcium sensitivities
C. Cell-surface receptors
D. Motor end plates
E. Subneural cleft architecture
ANSWER: C
3. The effect of autonomic nerve stimulation on smooth muscle contractile
behavior depends on the types of receptors on the smooth muscle fibers. Which
of the following best describes the effects of acetylcholine and norepinephrine on
various types of smooth muscle?
Acetylcholine Norepinephrine
A. Contraction only Relaxation or contraction
B. Contraction only Relaxation only
C. Relaxation only Contraction only
D. Relaxation only Relaxation only
E. Relaxation or contraction Contraction only
F. Relaxation or contraction Relaxation or contraction
ANSWER: F
4. Unlike skeletal muscle, the contraction of smooth muscle requires which
of the following?
A. Activation of ryanodine receptors
B. Phosphorylation of myosin light chains
C. The presence of intracellular calcium
D. Troponin binding of calcium
E. Voltage activation of dihydropyridine receptors
ANSWER: B
5. Which of the following factors are important for initiating smooth muscle
contractions?
Hormones Paracrine factors Autonomic nervous
system
A. No No No
B. No No Yes
C. Yes No Yes
D. Yes
E. Yes
Yes
Yes
No
Yes
ANSWER: E
6. The sensitivity of the smooth muscle contractile apparatus to calcium can
increase. This increase in calcium sensitivity can be attributed to a decrease in
the levels of which of the following substances?
A. Actin
B. Adenosine triphosphate (ATP)
C. Calcium-calmodulin complex
D. Calmodulin
E. Myosin light chain phosphatase
ANSWER: E
7. Which of the following is the correct temporal order of events for the smooth
muscle?
A. Myosin light chain (MLC) dephosphorylation, contraction,
relaxation, MLC phosphorylation
B. MLC dephosphorylation, contraction, MLC phosphorylation, relaxation
C. MLC dephosphorylation, relaxation, MLC phosphorylation, relaxation
D. MLC phosphorylation, contraction, MLC dephosphorylation, relaxation
E. MLC phosphorylation, contraction, relaxation, MLC dephosphorylation
F. MLC phosphorylation, MLC dephosphorylation, contraction, relaxation
ANSWER: D
Chapter 9. Cardiac Muscle; The Heart as a Pump and Function of the
Heart Valves
Questions 1-3: A fifty-year-old man has a body weight of 110 kg, resting heart rate
is 90 beats/minute, arterial pressure is 120/70 mmHg, arterial hematocrit is 35%
and body temperature is normal. His pressure-volume diagram of the left
ventricle is shown below.
1. What is his resting cardiac output in ml/minute?
A. 900
B. 8,000
C. 9,000
D. 10,000
E. 15,000
ANSWER: C
2. What event occurs at point D in the man’s pressure-volume relationship?
A. A-V valves close
B. Aortic valve opens
C. Aortic valve closes
D. A-V valves open
E. Third heart sound
ANSWER: C
3. What is his ventricular ejection
fraction?A. 33%
B. 50%
C. 60%
D. 67%
E. 80%
ANSWER:
D
4. Which of the following will result in spastic contraction of the heart?
A. Excess calcium ions in the blood
B. Excess potassium ions in the blood
C. Excess sodium ions in the blood
D. Decreased sympathetic stimulation
E. Decreased norepinephrine concentration in the blood
ANSWER: A
Questions 5-7: A sixty-year-old woman has a body weight of 110 kg, resting cardiac
output is 9,000 ml/minute, arterial pressure is 90/50 mmHg, arterial hematocrit is
25% and body temperature is normal. Her pressure-volume diagram of the left
ventricle is shown below.
5. What is her resting heart rate in beats/minute?
A. 70
B. 80
C. 90
D.
100 E.
110
ANSWER: C
6. What event occurs at point A in the woman’s pressure-volume relationship?
A. A-V valves close
B. Aortic valve opens
C. Aortic valve closes
D. A-V valves open
E. Third heart sound
ANSWER: D
7. If the woman is given a 500 ml whole blood transfusion over a 15 minute
period, her ventricular preload will increase. One minute after the transfusion is
complete, what will the new stroke volume be assuming her heart is normal?
A. 70 ml
B. 80 ml
C. 90 ml
D. 100 ml
E. 120 ml
ANSWER: A
Questions 8-9: A fifty-year-old man has a body weight of 110 kg, resting heart rate
is 90 beats/minute, arterial pressure is 120/70 mmHg, arterial hematocrit is 35%
and body temperature is normal. His pressure-volume diagram of the left
ventricle is shown below.
8. When does the third heart sound occur in the ventricular pressure-
volume relationship?
A. At point D
B. Between point A and point B
C. Between point B and point C
D. Between point C and point D
E. Between point D and point A
ANSWER: B
9. When does the second heart sound occur in the ventricular pressure-
volume relationship?
A. At point D
B. Between point A and point B
C. Between point B and point C
D. Between point C and point D
E. Between point D and point A
ANSWER: A
Chapter 10. Rhythmical Excitation of the Heart
Test Bank
1. If the sinus node stops discharging, what is the expected heart rate (in
beats/minute) if the A-V node takes over as the cardiac pacemaker?
A. 15
B. 20
C. 35
D. 50
E. 72
ANSWE
R: D
2. If the S-A node discharges at 0.00 seconds, when will the action potential
normally arrive at the A-V node?
A. 0.03 seconds
B. 0.09 seconds
C. 0.12 seconds
D. 0.16 seconds
E. 0.80 seconds
ANSWER: A
3. Which of the following conditions at the S-A node will cause a decrease in heart
rate?
A. Increased sodium permeability
B. Decreased acetylcholine levels
C. Increased norepinephrine levels
D. Increased potassium permeability
E. Increased calcium permeability
ANSWER: D
4. In which phase of the ventricular muscle action potential is the
potassium permeability the highest?
A. 0
B. 1
C. 2
D. 3
E. 4
ANSWE
R: D
5. If the S-A node discharges at 0.00 seconds, when will the action potential
normally arrive at the A-V bundle?
A. 0.80 seconds
B. 0.16 seconds
C. 0.12 seconds
D. 0.09 seconds
E. 0.03 seconds
ANSWER: C
6. A patient entered the local emergency room and had an EKG taken. The
attending physician stated that the patient had an A-V nodal rhythm. What is
the likely heart rate?
A. 30/min
B. 50/min
C. 65/min
D. 75/min
E. 85/min
ANSWER: B
7. Which of the following conditions at the S-A node will cause a decrease in heart
rate?
A. Increased sodium permeability
B. Increased acetylcholine levels
C. Increased norepinephrine levels
D. Decreased potassium permeability
E. Increased calcium permeability
ANSWER: B
8. Which of the following conditions at the A-V node will cause a decrease in
heart rate?
A. Increased sodium permeability
B. Decreased acetylcholine levels
C. Increased norepinephrine levels
D. Increased potassium permeability
E. Increased calcium permeability
ANSWER: D
9. If the S-A node discharges at 0.00 seconds, when will the action potential
normally arrive at the ventricular septum?
A. 0.22 seconds
B. 0.16 seconds
C. 0.12 seconds
D. 0.09 seconds
E. 0.03 seconds
ANSWER: B
10. What is the normal total delay of the cardiac impulse in the A-V node + bundle?
A. 0.22 seconds
B. 0.18 seconds
C. 0.16 seconds
D. 0.13 seconds
E. 0.09 seconds
ANSWER: D
11. Which of the following are caused by acetylcholine?
A. Hyperpolarization of the S-A node
B. Depolarization of the A-V node
C. Decreased permeability of the S-A node to potassium ions
D. Increased heart rate
E. Increased permeability of the cardiac muscle to calcium ions
ANSWER: A
12. If the sinus node stops discharging, what is the expected heart rate (in
beats/minute) if the A-V node takes over as the cardiac pacemaker?
A. 5
B. 30
C. 50
D. 70
E. 90
ANSWE
R: C
13. Sympathetic stimulation of the heart:
A. Releases acetylcholine at the sympathetic endings
B. Decreases sinus nodal discharge rate
C. Decreases excitability of the heart
D. Releases norepinephrine at the sympathetic endings
E. Decreases cardiac contractility
ANSWER: D
Chapter 11. The Normal Electrocardiogram
Test Bank
1. The Q-T interval is the time that approximates the time of ventricular
contraction. The normal Q-T interval is:
A. 0.04 seconds
B. 0.08 seconds
C. 0.16 seconds
D. 0.20 seconds
E. 0.35 seconds
ANSWER: E
Questions 2-3: A seventy-year old female had an electrocardiogram performed at
her annual checkup, and the following lead II recording was made.
2. What is her heart rate in beats/minute?
A. 68
B. 78
C. 84
D. 94
E. 104
ANSWER
: A
3. According to Einthoven’s Law, if the QRS voltage in her lead III is 0.5 mV, what
is the QRS voltage in Lead I?
A. 0.05 mV
B. 0.50 mV
C. 1.05 mV
D. 1.25 mV
E. 2.05 mV
ANSWER: B
4. The P-R interval is the time between the beginning of atrial
depolarization and ventricular depolarization. The normal P-R interval
is:
A. 0.04 seconds
B. 0.08 seconds
C. 0.16 seconds
D. 0.20 seconds
E. 0.35 seconds
ANSWER: C
5. When recording lead I on an EKG, the right arm is the negative electrode and
the positive electrode is the:
A. Left arm
B. Left leg
C. Right leg
D. Left arm + left leg
E. Right arm + left leg
ANSWER: A
Chapter 12. Electrocardiographic Interpretation of Cardiac
Muscle and Coronary Blood Flow Abnormalities: Vectorial
Analysis
Test Bank
1. A ventricular depolarization wave when traveling 210 in the frontal plane will
cause a large positive deflection in:
A. aVR
B. aVL
C. Lead II
D. Lead III
E. aVF
ANSWER:
A
Questions 2-4: A 60-year-old woman had the following EKG recorded at a local
emergency room following an automobile accident. Her weight was 70 kg and her
aortic blood pressure was 140/80 mmHg.
2. What is the mean electrical axis calculated from standard leads I, II, and III shown
in
the woman’s EKG?
A. −90
B. −50
C. −12
D. +105
E. +170
ANSWER:
D
3. What is the heart rate using lead I for the calculation?
A. 70
B. 88
C. 100
D. 112
E. 148
ANSWER
: A
4. What is her likely diagnosis?
A. Mitral valve stenosis
B. Left bundle branch block
C. Pulmonary valve stenosis
D. Right bundle branch block
E. Left ventricular hypertrophy
ANSWER: D
5. A seventy-five year-old woman came into a local emergency room complaining
of chest pain and shortness of breath. Blood samples were taken for cardiac
enzyme measurements, and an EKG was quickly taken. Based on the EKG
tracing shown below, what is the likely diagnosis?
A. Left bundle branch block
B. Acute anterior wall infarction in the base of the heart
C. Acute anterior wall infarction in the apex of the heart
D. Acute posterior infarction in the base of the heart
E. Acute posterior infarction in the apex of the heart
ANSWER: D
6. A ventricular depolarization wave when traveling 90 in the frontal plane will
cause a large positive deflection in:
A. aVR
B. aVL
C. Lead II
D. Lead III
E. aVF
ANSWER: A
Questions 7-9: A 70-year-old man had the following EKG recorded at his doctor’s
office. His weight was 80 kg and his blood pressure was 160/90 mmHg. He
complained that he gets tired easily.
7. What is the mean electrical axis calculated from standard leads I, II, and III shown
in
the man’s EKG?
A. −90
B. −60
C. −15
D. +60
E. +170
ANSWER: C
8. What is the heart rate using lead I for the calculation?
A. 75
B. 88
C. 100
D. 112
E. 148
ANSWER
: B
9. What is his likely diagnosis?
A. Tricuspid valve stenosis
B. Complete left bundle branch block
C. Pulmonary valve stenosis
D. Pulmonary valve regurgitation
E. Left ventricular hypertrophy
ANSWER: E
10. A seventy-year-old woman came into the local emergency room complaining of
chest pain and shortness of breath. Blood samples were taken for cardiac
enzyme measurements, and an EKG was quickly taken. Based on the EKG
tracing shown below, what is the likely diagnosis?
A. Left bundle branch block
B. Acute anterior wall infarction in the left ventricle
C. Acute anterior wall infarction in the right ventricle
D. Acute posterior infarction in the left ventricle
E. Acute posterior infarction in the right ventricle
ANSWER: B
Questions 11-12: A 65-year-old man went for an annual checkup and had the
following EKG recorded at his physician's office.
11. What is the mean electrical axis calculated from standard leads I, II, and III
shown in his EKG?
A. −50
B. −15
C. 0
D. +15
E. +50
ANSWER: B
12. What is the most likely diagnosis?
A. Right ventricular hypertrophy
B. Left bundle branch block
C. Pulmonary valve insufficiency
D. Right bundle branch block
E. Systemic hypertension
ANSWER: E
Questions 13-14. A 50-year-old woman was admitted to a local emergency room
following a motorcycle accident, and the following EKG was taken.
13. What is her heart rate? Use lead I for the calculation.
A. 56
B. 66
C. 76
D. 103
E. 152
ANSWER
: D
14. What is the most likely diagnosis?
A. Aortic valve insufficiency
B. Left bundle branch block
C. Pulmonary valve stenosis
D. Right bundle branch block
E. Systemic hypertension
ANSWER: C
15. During the TP interval in an electrocardiogram of a patient with a damaged
cardiac muscle:
A. The entire heart is depolarized
B. All the heart is depolarized except for the damaged cardiac muscle
C. About half the heart is depolarized
D. All of the heart is repolarized
E. All of the heart is repolarized except for the damaged cardiac muscle
ANSWER: E
16. A 30-year-old woman had an EKG taken, and her QRS complex had a width of
0.20 seconds, her T wave was inverted in lead I, and the R wave had a large
negative deflection in lead III. What is her diagnosis?
A. Right ventricular hypertrophy
B. Left bundle branch block
C. Pulmonary valve stenosis
D. Right bundle branch block
E. Left ventricular hypertrophy
ANSWER: E
17. A ventricular depolarization wave when traveling −30 in the frontal plane will
cause a
large positive deflection in:
A. aVR
B. aVL
C. Lead II
D. Lead III
E. aVF
ANSWER:
B
Questions 18-19: A 6-year-old boy had the following EKG recorded at his physician's
office. He had little energy, and during light exercise his respiratory would increase
rapidly.
18. What is his mean electrical axis calculated from standard leads I, II, and III
shown in his EKG?
A. −100
B. −20
C. 0
D.
+90
E. +170
ANSWER: E
19. What is his diagnosis?
A. Left ventricular hypertrophy
B. Left bundle branch block
C. Tricuspid stenosis
D. Right bundle branch block
E. Tetralogy of Fallot
ANSWER: E
E
20. Mr. Smith had his EKG measured at UMC, but they lost his records. The EKG
technician remembered that the QRS deflection was large and negative in Lead
aVF and 0 in Lead I. His mean electrical axis in the frontal plane is
A. 90
B. 60
C. 0
D.
−60 E.
−90
ANSWER
: E
Chapter 13.Cardiac Arrhythmias and Their Electrocardiographic
Interpretation
1. A forty-year-old man reported to his cardiologist that he had unusual feelings in
his chest after drinking his fourth cup of coffee in the morning. The cardiologist
recorded the following EKG during an exercise stress test. Leads II and III are
shown. What is the likely diagnosis?
A. Premature contraction originating in the atrium
B. Premature contraction originating high in the A-V node
C. Premature contraction originating low in the A-V node
D. Premature contraction originating in the apex of the ventricle
E. Premature contraction originating in the base of the ventricle
ANSWER: EE
2. A ninety-year-old man had an EKG taken at his local doctor’s office, and the
diagnosis was atrial fibrillation. Which of the following statements are likely
conditions in someone with atrial fibrillation?
A. Ventricular fibrillation normally accompanies atrial fibrillation
B. The P waves of the EKG are strong
C. The rate of ventricular contraction is irregular and fast
D. The atrial “a” wave is normal
E. The atria have a smaller volume than normal
ANSWER: C
3. Circus movements in the ventricle can lead to ventricular fibrillation. Which of
the following conditions in the ventricular muscle will increase the tendency
for circus movements?
A. Decreased refractory period
B. Low extracellular potassium concentration
C. Increased refractory period
D. Shorter conduction pathway (decreased ventricular volume)
E. Increase in parasympathetic impulses to the heart
ANSWER: A
4. A 55-year-old woman had the following EKG recorded at her annual physical.
She reported to her doctor that sometimes her pulse rate feels strong and at
other times it feels weak. Standard limb lead II is shown.
What is her likely diagnosis?
A. Atrial paroxysmal tachycardia
B. First degree A-V block
C. Second degree A-V block, Mobitz type I (Wenckebach)
D. Second degree A-V block, Mobitz type II
E. Third degree A-V block
ANSWER: D
5. A fifty-year-old man has been having fainting “spells” for about two weeks.
During the fainting episodes, his EKG shows a ventricular beat of 25 per
minute and 100 P waves per minute. After about 30 seconds of fainting a
normal sinus rhythm reoccurs. What is his likely diagnosis?
A. Atrial flutter
B. First degree A-V block
C. Second degree A-V block
D. Third degree A-V block
E. Stokes-Adams syndrome
ANSWER: E
6. A 65-year-old woman had the following EKG recorded at her annual physical.
She reported to her doctor that sometimes her pulse rate feels a little irregular.
Standard limb lead III is shown. What is her likely diagnosis?
A. Atrial paroxysmal tachycardia
B. First degree A-V block
C. Second degree A-V block, Mobitz type I (Wenckebach)
D. Normal sinus rhythm
E. Third degree A-V block
ANSWER: EE
7. Circus movements in the ventricle can lead to ventricular fibrillation. Which of
the following conditions in the ventricular muscle will increase the tendency
for circus movements?
A. Decreased conduction velocity
B. Low extracellular potassium concentration
C. Increased refractory period
D. Shorter conduction pathway (decreased ventricular volume)
E. Increase in parasympathetic impulses to the heart
ANSWER: AA
8. Which of the following statements about a premature atrial contraction is true?
A. The radial artery pulse associated with the premature contraction will be weak
B. Stroke volume associated with the premature contraction will be increased
C. The P wave is never seen
D. The probability of these premature contractions occurring is decreased in
people with a large caffeine intake
E. It causes the QRS interval to be lengthened.
ANSW
ER: A
A
9. A 65-year-old man reported some unusual sensations in his chest. His blood
pressure is 135/85 mmHg, and he weighs 200 pounds. He had an EKG taken,
and it is shown below. What is his likely diagnosis?
A. Atrial paroxysmal tachycardia
B. First degree A-V block
C. Second degree A-V block
D. Premature atrial contractions
E. Premature ventricular contractions
IF YOU WANT THIS TEST BANK OR
SOLUTION MANUAL EMAIL ME
rightmanforbloodline1@gmail.com TO
RECEIVE ALL CHAPTERS IN PDF FORMAT
IF YOU WANT THIS TEST BANK OR
SOLUTION MANUAL EMAIL ME
rightmanforbloodline1@gmail.com TO
RECEIVE ALL CHAPTERS IN PDF FORMAT

More Related Content

What's hot

Mechanism of blood flow control-I
Mechanism of blood flow control-IMechanism of blood flow control-I
Mechanism of blood flow control-Irashidrmc
 
Excitable Tissues, Resting Membrane Potential & Action.pptx
Excitable Tissues, Resting Membrane Potential & Action.pptxExcitable Tissues, Resting Membrane Potential & Action.pptx
Excitable Tissues, Resting Membrane Potential & Action.pptxAnwar Siddiqui
 
Anatomy & Physiology Lecture Notes - Ch. 3 cells - part 1
Anatomy & Physiology Lecture Notes - Ch. 3 cells - part 1Anatomy & Physiology Lecture Notes - Ch. 3 cells - part 1
Anatomy & Physiology Lecture Notes - Ch. 3 cells - part 1Ammedicine Medicine
 
Nervous system
Nervous systemNervous system
Nervous systemsom allul
 
A.3 endocrine system & glands
A.3   endocrine system & glandsA.3   endocrine system & glands
A.3 endocrine system & glandsserenaasya
 
Physiology of gas exchange
Physiology of gas exchangePhysiology of gas exchange
Physiology of gas exchangeHimanshu Jangid
 
Nervous System Introduction and Central Nervous System
Nervous System Introduction and Central Nervous SystemNervous System Introduction and Central Nervous System
Nervous System Introduction and Central Nervous SystemRHSHealthScience
 
Central nervous-system-09-10
Central nervous-system-09-10Central nervous-system-09-10
Central nervous-system-09-10Isyafiq qamaal
 
Presentation on anatomy of heart ,coronary circulation ,mi and ecg interpreta...
Presentation on anatomy of heart ,coronary circulation ,mi and ecg interpreta...Presentation on anatomy of heart ,coronary circulation ,mi and ecg interpreta...
Presentation on anatomy of heart ,coronary circulation ,mi and ecg interpreta...febin rony
 

What's hot (20)

Mechanism of blood flow control-I
Mechanism of blood flow control-IMechanism of blood flow control-I
Mechanism of blood flow control-I
 
Regulation of Respiration
Regulation of RespirationRegulation of Respiration
Regulation of Respiration
 
CNS MCQs
CNS MCQsCNS MCQs
CNS MCQs
 
Excitable Tissues, Resting Membrane Potential & Action.pptx
Excitable Tissues, Resting Membrane Potential & Action.pptxExcitable Tissues, Resting Membrane Potential & Action.pptx
Excitable Tissues, Resting Membrane Potential & Action.pptx
 
Cardiovascular system
Cardiovascular systemCardiovascular system
Cardiovascular system
 
Neuroglia, syapse
Neuroglia, syapseNeuroglia, syapse
Neuroglia, syapse
 
Structure of neuron
Structure of neuronStructure of neuron
Structure of neuron
 
Nervous coordination
Nervous coordinationNervous coordination
Nervous coordination
 
Nervous tissue histology
Nervous tissue  histology Nervous tissue  histology
Nervous tissue histology
 
Anatomy & Physiology Lecture Notes - Ch. 3 cells - part 1
Anatomy & Physiology Lecture Notes - Ch. 3 cells - part 1Anatomy & Physiology Lecture Notes - Ch. 3 cells - part 1
Anatomy & Physiology Lecture Notes - Ch. 3 cells - part 1
 
Nervous system
Nervous systemNervous system
Nervous system
 
Heart
HeartHeart
Heart
 
A.3 endocrine system & glands
A.3   endocrine system & glandsA.3   endocrine system & glands
A.3 endocrine system & glands
 
3. homeostasis
3. homeostasis3. homeostasis
3. homeostasis
 
General physiology
General physiologyGeneral physiology
General physiology
 
Physiology of gas exchange
Physiology of gas exchangePhysiology of gas exchange
Physiology of gas exchange
 
Compliance of lung
Compliance of lungCompliance of lung
Compliance of lung
 
Nervous System Introduction and Central Nervous System
Nervous System Introduction and Central Nervous SystemNervous System Introduction and Central Nervous System
Nervous System Introduction and Central Nervous System
 
Central nervous-system-09-10
Central nervous-system-09-10Central nervous-system-09-10
Central nervous-system-09-10
 
Presentation on anatomy of heart ,coronary circulation ,mi and ecg interpreta...
Presentation on anatomy of heart ,coronary circulation ,mi and ecg interpreta...Presentation on anatomy of heart ,coronary circulation ,mi and ecg interpreta...
Presentation on anatomy of heart ,coronary circulation ,mi and ecg interpreta...
 

Similar to TEST BANK For Guyton and Hall Textbook of Medical Physiology, 14th Edition by John E. Hall; Michael E. Hall, Verified Chapters 1 - 86, Complete Newest Version.pdf

Guyton table of_content
Guyton table of_contentGuyton table of_content
Guyton table of_contentLogique Brain
 
DTCM1134 Introduction to Physiology.ppt
DTCM1134 Introduction to Physiology.pptDTCM1134 Introduction to Physiology.ppt
DTCM1134 Introduction to Physiology.pptchinvoonkin
 
Latest syllabus for UG students by the Dental Council of India
Latest syllabus for UG students by the Dental Council of IndiaLatest syllabus for UG students by the Dental Council of India
Latest syllabus for UG students by the Dental Council of IndiaDr. Anshuman Bag
 
Physiology in mbbs
Physiology in mbbsPhysiology in mbbs
Physiology in mbbsRaghu Veer
 
introduction to Anatomy terminology
introduction to Anatomy terminologyintroduction to Anatomy terminology
introduction to Anatomy terminologyDrDerick
 
Blood properties, components and composition
Blood properties, components and compositionBlood properties, components and composition
Blood properties, components and compositionVamsi kumar
 
HAP-I, Syllabus for Theory and Practical
HAP-I, Syllabus for Theory and PracticalHAP-I, Syllabus for Theory and Practical
HAP-I, Syllabus for Theory and PracticalAudumbar Mali
 
Anatomy and physiology
Anatomy and physiologyAnatomy and physiology
Anatomy and physiologyIndia™
 
Topic 6 4, 6 5, 6 6
Topic 6 4, 6 5, 6 6Topic 6 4, 6 5, 6 6
Topic 6 4, 6 5, 6 6bobalong
 
Hematology learning guide
Hematology learning guide Hematology learning guide
Hematology learning guide Fidaa Jaafrah
 
In this lab, you will observe diagrams that show the major component.docx
In this lab, you will observe diagrams that show the major component.docxIn this lab, you will observe diagrams that show the major component.docx
In this lab, you will observe diagrams that show the major component.docxwiddowsonerica
 
Chapter 1
Chapter 1 Chapter 1
Chapter 1 bholmes
 
Krok 1 - 2014 (Physiology)
Krok 1 - 2014 (Physiology)Krok 1 - 2014 (Physiology)
Krok 1 - 2014 (Physiology)Eneutron
 

Similar to TEST BANK For Guyton and Hall Textbook of Medical Physiology, 14th Edition by John E. Hall; Michael E. Hall, Verified Chapters 1 - 86, Complete Newest Version.pdf (20)

Guyton table of_content
Guyton table of_contentGuyton table of_content
Guyton table of_content
 
DTCM1134 Introduction to Physiology.ppt
DTCM1134 Introduction to Physiology.pptDTCM1134 Introduction to Physiology.ppt
DTCM1134 Introduction to Physiology.ppt
 
Latest syllabus for UG students by the Dental Council of India
Latest syllabus for UG students by the Dental Council of IndiaLatest syllabus for UG students by the Dental Council of India
Latest syllabus for UG students by the Dental Council of India
 
Presentation 1.pptx
Presentation 1.pptxPresentation 1.pptx
Presentation 1.pptx
 
Anatomy first-day
Anatomy first-dayAnatomy first-day
Anatomy first-day
 
Physiology in mbbs
Physiology in mbbsPhysiology in mbbs
Physiology in mbbs
 
introduction to Anatomy terminology
introduction to Anatomy terminologyintroduction to Anatomy terminology
introduction to Anatomy terminology
 
Blood properties, components and composition
Blood properties, components and compositionBlood properties, components and composition
Blood properties, components and composition
 
Tissues notes
Tissues notesTissues notes
Tissues notes
 
HAP-I, Syllabus for Theory and Practical
HAP-I, Syllabus for Theory and PracticalHAP-I, Syllabus for Theory and Practical
HAP-I, Syllabus for Theory and Practical
 
Anatomy and physiology
Anatomy and physiologyAnatomy and physiology
Anatomy and physiology
 
Topic 6 4, 6 5, 6 6
Topic 6 4, 6 5, 6 6Topic 6 4, 6 5, 6 6
Topic 6 4, 6 5, 6 6
 
Hematology learning guide 1
Hematology learning guide 1 Hematology learning guide 1
Hematology learning guide 1
 
Hematology learning guide
Hematology learning guide Hematology learning guide
Hematology learning guide
 
Pagine da urso scientific basis sport training
Pagine da urso scientific basis sport trainingPagine da urso scientific basis sport training
Pagine da urso scientific basis sport training
 
In this lab, you will observe diagrams that show the major component.docx
In this lab, you will observe diagrams that show the major component.docxIn this lab, you will observe diagrams that show the major component.docx
In this lab, you will observe diagrams that show the major component.docx
 
CH 1 Cell Physiology.pdf
CH  1 Cell Physiology.pdfCH  1 Cell Physiology.pdf
CH 1 Cell Physiology.pdf
 
Pgy 2020 course handbook 2014 1
Pgy 2020 course handbook 2014   1Pgy 2020 course handbook 2014   1
Pgy 2020 course handbook 2014 1
 
Chapter 1
Chapter 1 Chapter 1
Chapter 1
 
Krok 1 - 2014 (Physiology)
Krok 1 - 2014 (Physiology)Krok 1 - 2014 (Physiology)
Krok 1 - 2014 (Physiology)
 

More from rightmanforbloodline

TEST BANK For Robbins & Kumar Basic Pathology, 11th Edition by Vinay Kumar, A...
TEST BANK For Robbins & Kumar Basic Pathology, 11th Edition by Vinay Kumar, A...TEST BANK For Robbins & Kumar Basic Pathology, 11th Edition by Vinay Kumar, A...
TEST BANK For Robbins & Kumar Basic Pathology, 11th Edition by Vinay Kumar, A...rightmanforbloodline
 
Test Bank -Medical-Surgical Nursing Concepts for Interprofessional Collaborat...
Test Bank -Medical-Surgical Nursing Concepts for Interprofessional Collaborat...Test Bank -Medical-Surgical Nursing Concepts for Interprofessional Collaborat...
Test Bank -Medical-Surgical Nursing Concepts for Interprofessional Collaborat...rightmanforbloodline
 
TEST BANK For Success in Practical Vocational Nursing 10th Edition by Carrol ...
TEST BANK For Success in Practical Vocational Nursing 10th Edition by Carrol ...TEST BANK For Success in Practical Vocational Nursing 10th Edition by Carrol ...
TEST BANK For Success in Practical Vocational Nursing 10th Edition by Carrol ...rightmanforbloodline
 
TEST BANK For Rau’s Respiratory Care Pharmacology, 10th Edition by Gardenhire...
TEST BANK For Rau’s Respiratory Care Pharmacology, 10th Edition by Gardenhire...TEST BANK For Rau’s Respiratory Care Pharmacology, 10th Edition by Gardenhire...
TEST BANK For Rau’s Respiratory Care Pharmacology, 10th Edition by Gardenhire...rightmanforbloodline
 
TEST BANK For Principles of Pediatric Nursing Caring for Children, 8th Editio...
TEST BANK For Principles of Pediatric Nursing Caring for Children, 8th Editio...TEST BANK For Principles of Pediatric Nursing Caring for Children, 8th Editio...
TEST BANK For Principles of Pediatric Nursing Caring for Children, 8th Editio...rightmanforbloodline
 
TEST BANK For Principles of Anatomy and Physiology, 16th Edition by Gerard J....
TEST BANK For Principles of Anatomy and Physiology, 16th Edition by Gerard J....TEST BANK For Principles of Anatomy and Physiology, 16th Edition by Gerard J....
TEST BANK For Principles of Anatomy and Physiology, 16th Edition by Gerard J....rightmanforbloodline
 
TEST BANK For Primary Care Psychiatry, 2nd Edition by Robert McCarron, Glen X...
TEST BANK For Primary Care Psychiatry, 2nd Edition by Robert McCarron, Glen X...TEST BANK For Primary Care Psychiatry, 2nd Edition by Robert McCarron, Glen X...
TEST BANK For Primary Care Psychiatry, 2nd Edition by Robert McCarron, Glen X...rightmanforbloodline
 
TEST BANK For Little and Falace's Dental Management of the Medically Compromi...
TEST BANK For Little and Falace's Dental Management of the Medically Compromi...TEST BANK For Little and Falace's Dental Management of the Medically Compromi...
TEST BANK For Little and Falace's Dental Management of the Medically Compromi...rightmanforbloodline
 
TEST BANK For Porth's Essentials of Pathophysiology, 5th Edition by Tommie L ...
TEST BANK For Porth's Essentials of Pathophysiology, 5th Edition by Tommie L ...TEST BANK For Porth's Essentials of Pathophysiology, 5th Edition by Tommie L ...
TEST BANK For Porth's Essentials of Pathophysiology, 5th Edition by Tommie L ...rightmanforbloodline
 
TEST BANK For Nursing Leadership, Management, and Professional Practice for t...
TEST BANK For Nursing Leadership, Management, and Professional Practice for t...TEST BANK For Nursing Leadership, Management, and Professional Practice for t...
TEST BANK For Nursing Leadership, Management, and Professional Practice for t...rightmanforbloodline
 
Solution Manual For Financial Statement Analysis, 13th Edition By Charles H. ...
Solution Manual For Financial Statement Analysis, 13th Edition By Charles H. ...Solution Manual For Financial Statement Analysis, 13th Edition By Charles H. ...
Solution Manual For Financial Statement Analysis, 13th Edition By Charles H. ...rightmanforbloodline
 
Solution manual for managerial accounting 8th edition by john wild ken shaw b...
Solution manual for managerial accounting 8th edition by john wild ken shaw b...Solution manual for managerial accounting 8th edition by john wild ken shaw b...
Solution manual for managerial accounting 8th edition by john wild ken shaw b...rightmanforbloodline
 
TEST BANK For Leddy & Pepper’s Professional Nursing, 10th Edition by Lucy Hoo...
TEST BANK For Leddy & Pepper’s Professional Nursing, 10th Edition by Lucy Hoo...TEST BANK For Leddy & Pepper’s Professional Nursing, 10th Edition by Lucy Hoo...
TEST BANK For Leddy & Pepper’s Professional Nursing, 10th Edition by Lucy Hoo...rightmanforbloodline
 
TEST BANK For Gynecologic Health Care With an Introduction to Prenatal and Po...
TEST BANK For Gynecologic Health Care With an Introduction to Prenatal and Po...TEST BANK For Gynecologic Health Care With an Introduction to Prenatal and Po...
TEST BANK For Gynecologic Health Care With an Introduction to Prenatal and Po...rightmanforbloodline
 
TEST BANK For Growth and Development Across the Lifespan, 3rd Edition By Glor...
TEST BANK For Growth and Development Across the Lifespan, 3rd Edition By Glor...TEST BANK For Growth and Development Across the Lifespan, 3rd Edition By Glor...
TEST BANK For Growth and Development Across the Lifespan, 3rd Edition By Glor...rightmanforbloodline
 
Solution Manual For Federal Tax Research, 13th Edition by Roby Sawyers, Steve...
Solution Manual For Federal Tax Research, 13th Edition by Roby Sawyers, Steve...Solution Manual For Federal Tax Research, 13th Edition by Roby Sawyers, Steve...
Solution Manual For Federal Tax Research, 13th Edition by Roby Sawyers, Steve...rightmanforbloodline
 
Global business today global 8th edition hill test bank.docx
Global business today global 8th edition hill test bank.docxGlobal business today global 8th edition hill test bank.docx
Global business today global 8th edition hill test bank.docxrightmanforbloodline
 
TEST BANK For Accounting Information Systems 4th Edition by Vernon Richardson...
TEST BANK For Accounting Information Systems 4th Edition by Vernon Richardson...TEST BANK For Accounting Information Systems 4th Edition by Vernon Richardson...
TEST BANK For Accounting Information Systems 4th Edition by Vernon Richardson...rightmanforbloodline
 
TEST BANKS For Family Practice Guidelines, 6th Edition by Jill C. Cash; Chery...
TEST BANKS For Family Practice Guidelines, 6th Edition by Jill C. Cash; Chery...TEST BANKS For Family Practice Guidelines, 6th Edition by Jill C. Cash; Chery...
TEST BANKS For Family Practice Guidelines, 6th Edition by Jill C. Cash; Chery...rightmanforbloodline
 
TEST BANK For Family Practice Guidelines, 5th Edition by Jill C. Cash; Cheryl...
TEST BANK For Family Practice Guidelines, 5th Edition by Jill C. Cash; Cheryl...TEST BANK For Family Practice Guidelines, 5th Edition by Jill C. Cash; Cheryl...
TEST BANK For Family Practice Guidelines, 5th Edition by Jill C. Cash; Cheryl...rightmanforbloodline
 

More from rightmanforbloodline (20)

TEST BANK For Robbins & Kumar Basic Pathology, 11th Edition by Vinay Kumar, A...
TEST BANK For Robbins & Kumar Basic Pathology, 11th Edition by Vinay Kumar, A...TEST BANK For Robbins & Kumar Basic Pathology, 11th Edition by Vinay Kumar, A...
TEST BANK For Robbins & Kumar Basic Pathology, 11th Edition by Vinay Kumar, A...
 
Test Bank -Medical-Surgical Nursing Concepts for Interprofessional Collaborat...
Test Bank -Medical-Surgical Nursing Concepts for Interprofessional Collaborat...Test Bank -Medical-Surgical Nursing Concepts for Interprofessional Collaborat...
Test Bank -Medical-Surgical Nursing Concepts for Interprofessional Collaborat...
 
TEST BANK For Success in Practical Vocational Nursing 10th Edition by Carrol ...
TEST BANK For Success in Practical Vocational Nursing 10th Edition by Carrol ...TEST BANK For Success in Practical Vocational Nursing 10th Edition by Carrol ...
TEST BANK For Success in Practical Vocational Nursing 10th Edition by Carrol ...
 
TEST BANK For Rau’s Respiratory Care Pharmacology, 10th Edition by Gardenhire...
TEST BANK For Rau’s Respiratory Care Pharmacology, 10th Edition by Gardenhire...TEST BANK For Rau’s Respiratory Care Pharmacology, 10th Edition by Gardenhire...
TEST BANK For Rau’s Respiratory Care Pharmacology, 10th Edition by Gardenhire...
 
TEST BANK For Principles of Pediatric Nursing Caring for Children, 8th Editio...
TEST BANK For Principles of Pediatric Nursing Caring for Children, 8th Editio...TEST BANK For Principles of Pediatric Nursing Caring for Children, 8th Editio...
TEST BANK For Principles of Pediatric Nursing Caring for Children, 8th Editio...
 
TEST BANK For Principles of Anatomy and Physiology, 16th Edition by Gerard J....
TEST BANK For Principles of Anatomy and Physiology, 16th Edition by Gerard J....TEST BANK For Principles of Anatomy and Physiology, 16th Edition by Gerard J....
TEST BANK For Principles of Anatomy and Physiology, 16th Edition by Gerard J....
 
TEST BANK For Primary Care Psychiatry, 2nd Edition by Robert McCarron, Glen X...
TEST BANK For Primary Care Psychiatry, 2nd Edition by Robert McCarron, Glen X...TEST BANK For Primary Care Psychiatry, 2nd Edition by Robert McCarron, Glen X...
TEST BANK For Primary Care Psychiatry, 2nd Edition by Robert McCarron, Glen X...
 
TEST BANK For Little and Falace's Dental Management of the Medically Compromi...
TEST BANK For Little and Falace's Dental Management of the Medically Compromi...TEST BANK For Little and Falace's Dental Management of the Medically Compromi...
TEST BANK For Little and Falace's Dental Management of the Medically Compromi...
 
TEST BANK For Porth's Essentials of Pathophysiology, 5th Edition by Tommie L ...
TEST BANK For Porth's Essentials of Pathophysiology, 5th Edition by Tommie L ...TEST BANK For Porth's Essentials of Pathophysiology, 5th Edition by Tommie L ...
TEST BANK For Porth's Essentials of Pathophysiology, 5th Edition by Tommie L ...
 
TEST BANK For Nursing Leadership, Management, and Professional Practice for t...
TEST BANK For Nursing Leadership, Management, and Professional Practice for t...TEST BANK For Nursing Leadership, Management, and Professional Practice for t...
TEST BANK For Nursing Leadership, Management, and Professional Practice for t...
 
Solution Manual For Financial Statement Analysis, 13th Edition By Charles H. ...
Solution Manual For Financial Statement Analysis, 13th Edition By Charles H. ...Solution Manual For Financial Statement Analysis, 13th Edition By Charles H. ...
Solution Manual For Financial Statement Analysis, 13th Edition By Charles H. ...
 
Solution manual for managerial accounting 8th edition by john wild ken shaw b...
Solution manual for managerial accounting 8th edition by john wild ken shaw b...Solution manual for managerial accounting 8th edition by john wild ken shaw b...
Solution manual for managerial accounting 8th edition by john wild ken shaw b...
 
TEST BANK For Leddy & Pepper’s Professional Nursing, 10th Edition by Lucy Hoo...
TEST BANK For Leddy & Pepper’s Professional Nursing, 10th Edition by Lucy Hoo...TEST BANK For Leddy & Pepper’s Professional Nursing, 10th Edition by Lucy Hoo...
TEST BANK For Leddy & Pepper’s Professional Nursing, 10th Edition by Lucy Hoo...
 
TEST BANK For Gynecologic Health Care With an Introduction to Prenatal and Po...
TEST BANK For Gynecologic Health Care With an Introduction to Prenatal and Po...TEST BANK For Gynecologic Health Care With an Introduction to Prenatal and Po...
TEST BANK For Gynecologic Health Care With an Introduction to Prenatal and Po...
 
TEST BANK For Growth and Development Across the Lifespan, 3rd Edition By Glor...
TEST BANK For Growth and Development Across the Lifespan, 3rd Edition By Glor...TEST BANK For Growth and Development Across the Lifespan, 3rd Edition By Glor...
TEST BANK For Growth and Development Across the Lifespan, 3rd Edition By Glor...
 
Solution Manual For Federal Tax Research, 13th Edition by Roby Sawyers, Steve...
Solution Manual For Federal Tax Research, 13th Edition by Roby Sawyers, Steve...Solution Manual For Federal Tax Research, 13th Edition by Roby Sawyers, Steve...
Solution Manual For Federal Tax Research, 13th Edition by Roby Sawyers, Steve...
 
Global business today global 8th edition hill test bank.docx
Global business today global 8th edition hill test bank.docxGlobal business today global 8th edition hill test bank.docx
Global business today global 8th edition hill test bank.docx
 
TEST BANK For Accounting Information Systems 4th Edition by Vernon Richardson...
TEST BANK For Accounting Information Systems 4th Edition by Vernon Richardson...TEST BANK For Accounting Information Systems 4th Edition by Vernon Richardson...
TEST BANK For Accounting Information Systems 4th Edition by Vernon Richardson...
 
TEST BANKS For Family Practice Guidelines, 6th Edition by Jill C. Cash; Chery...
TEST BANKS For Family Practice Guidelines, 6th Edition by Jill C. Cash; Chery...TEST BANKS For Family Practice Guidelines, 6th Edition by Jill C. Cash; Chery...
TEST BANKS For Family Practice Guidelines, 6th Edition by Jill C. Cash; Chery...
 
TEST BANK For Family Practice Guidelines, 5th Edition by Jill C. Cash; Cheryl...
TEST BANK For Family Practice Guidelines, 5th Edition by Jill C. Cash; Cheryl...TEST BANK For Family Practice Guidelines, 5th Edition by Jill C. Cash; Cheryl...
TEST BANK For Family Practice Guidelines, 5th Edition by Jill C. Cash; Cheryl...
 

Recently uploaded

MALE REPRODUCTIVE TOXICITY STUDIES(Toxicokinetics).pptx
MALE REPRODUCTIVE TOXICITY STUDIES(Toxicokinetics).pptxMALE REPRODUCTIVE TOXICITY STUDIES(Toxicokinetics).pptx
MALE REPRODUCTIVE TOXICITY STUDIES(Toxicokinetics).pptxKhanSabit
 
Aptopadesha Pramana / Pariksha: The Verbal Testimony
Aptopadesha Pramana / Pariksha: The Verbal TestimonyAptopadesha Pramana / Pariksha: The Verbal Testimony
Aptopadesha Pramana / Pariksha: The Verbal TestimonyDr KHALID B.M
 
180-hour Power Capsules For Men In Ghana
180-hour Power Capsules For Men In Ghana180-hour Power Capsules For Men In Ghana
180-hour Power Capsules For Men In Ghanahealthwatchghana
 
DECIPHERING COMMON ECG FINDINGS IN ED.pptx
DECIPHERING COMMON ECG FINDINGS IN ED.pptxDECIPHERING COMMON ECG FINDINGS IN ED.pptx
DECIPHERING COMMON ECG FINDINGS IN ED.pptxdrwaque
 
𝕔ALL #𝕘IRLS Service in Jaipur %(8901183002)# Jaipur #𝕔ALL #𝕘IRLS
𝕔ALL #𝕘IRLS Service in Jaipur %(8901183002)#  Jaipur #𝕔ALL #𝕘IRLS𝕔ALL #𝕘IRLS Service in Jaipur %(8901183002)#  Jaipur #𝕔ALL #𝕘IRLS
𝕔ALL #𝕘IRLS Service in Jaipur %(8901183002)# Jaipur #𝕔ALL #𝕘IRLSlakehe2738
 
PT MANAGEMENT OF URINARY INCONTINENCE.pptx
PT MANAGEMENT OF URINARY INCONTINENCE.pptxPT MANAGEMENT OF URINARY INCONTINENCE.pptx
PT MANAGEMENT OF URINARY INCONTINENCE.pptxdrtabassum4
 
Antiplatelets in IHD, Dose Duration, DAPT vs SAPT
Antiplatelets in IHD, Dose Duration, DAPT vs SAPTAntiplatelets in IHD, Dose Duration, DAPT vs SAPT
Antiplatelets in IHD, Dose Duration, DAPT vs SAPTAkashGanganePatil1
 
End Feel -joint end feel - Normal and Abnormal end feel
End Feel -joint end feel - Normal and Abnormal end feelEnd Feel -joint end feel - Normal and Abnormal end feel
End Feel -joint end feel - Normal and Abnormal end feeldranji1
 
Relationship between vascular system disfunction, neurofluid flow and Alzheim...
Relationship between vascular system disfunction, neurofluid flow and Alzheim...Relationship between vascular system disfunction, neurofluid flow and Alzheim...
Relationship between vascular system disfunction, neurofluid flow and Alzheim...Catherine Liao
 
Gauri Gawande(9) Constipation Final.pptx
Gauri Gawande(9) Constipation Final.pptxGauri Gawande(9) Constipation Final.pptx
Gauri Gawande(9) Constipation Final.pptxgauripg8
 
Vaccines: A Powerful and Cost-Effective Tool Protecting Americans Against Dis...
Vaccines: A Powerful and Cost-Effective Tool Protecting Americans Against Dis...Vaccines: A Powerful and Cost-Effective Tool Protecting Americans Against Dis...
Vaccines: A Powerful and Cost-Effective Tool Protecting Americans Against Dis...PhRMA
 
In-service education (Nursing Mangement)
In-service education (Nursing Mangement)In-service education (Nursing Mangement)
In-service education (Nursing Mangement)Monika Kanwar
 
Retinal consideration in cataract surgery
Retinal consideration in cataract surgeryRetinal consideration in cataract surgery
Retinal consideration in cataract surgeryKafrELShiekh University
 
Prix Galien International 2024 Forum Program
Prix Galien International 2024 Forum ProgramPrix Galien International 2024 Forum Program
Prix Galien International 2024 Forum ProgramLevi Shapiro
 
Hemodialysis: Chapter 2, Extracorporeal Blood Circuit - Dr.Gawad
Hemodialysis: Chapter 2, Extracorporeal Blood Circuit - Dr.GawadHemodialysis: Chapter 2, Extracorporeal Blood Circuit - Dr.Gawad
Hemodialysis: Chapter 2, Extracorporeal Blood Circuit - Dr.GawadNephroTube - Dr.Gawad
 
Effects of vaping e-cigarettes on arterial health
Effects of vaping e-cigarettes on arterial healthEffects of vaping e-cigarettes on arterial health
Effects of vaping e-cigarettes on arterial healthCatherine Liao
 
Introducing VarSeq Dx as a Medical Device in the European Union
Introducing VarSeq Dx as a Medical Device in the European UnionIntroducing VarSeq Dx as a Medical Device in the European Union
Introducing VarSeq Dx as a Medical Device in the European UnionGolden Helix
 
Cardiovascular Physiology - Regulation of Cardiac Pumping
Cardiovascular Physiology - Regulation of Cardiac PumpingCardiovascular Physiology - Regulation of Cardiac Pumping
Cardiovascular Physiology - Regulation of Cardiac PumpingMedicoseAcademics
 
Final CAPNOCYTOPHAGA INFECTION by Gauri Gawande.pptx
Final CAPNOCYTOPHAGA INFECTION by Gauri Gawande.pptxFinal CAPNOCYTOPHAGA INFECTION by Gauri Gawande.pptx
Final CAPNOCYTOPHAGA INFECTION by Gauri Gawande.pptxgauripg8
 
Arterial health throughout cancer treatment and exercise rehabilitation in wo...
Arterial health throughout cancer treatment and exercise rehabilitation in wo...Arterial health throughout cancer treatment and exercise rehabilitation in wo...
Arterial health throughout cancer treatment and exercise rehabilitation in wo...Catherine Liao
 

Recently uploaded (20)

MALE REPRODUCTIVE TOXICITY STUDIES(Toxicokinetics).pptx
MALE REPRODUCTIVE TOXICITY STUDIES(Toxicokinetics).pptxMALE REPRODUCTIVE TOXICITY STUDIES(Toxicokinetics).pptx
MALE REPRODUCTIVE TOXICITY STUDIES(Toxicokinetics).pptx
 
Aptopadesha Pramana / Pariksha: The Verbal Testimony
Aptopadesha Pramana / Pariksha: The Verbal TestimonyAptopadesha Pramana / Pariksha: The Verbal Testimony
Aptopadesha Pramana / Pariksha: The Verbal Testimony
 
180-hour Power Capsules For Men In Ghana
180-hour Power Capsules For Men In Ghana180-hour Power Capsules For Men In Ghana
180-hour Power Capsules For Men In Ghana
 
DECIPHERING COMMON ECG FINDINGS IN ED.pptx
DECIPHERING COMMON ECG FINDINGS IN ED.pptxDECIPHERING COMMON ECG FINDINGS IN ED.pptx
DECIPHERING COMMON ECG FINDINGS IN ED.pptx
 
𝕔ALL #𝕘IRLS Service in Jaipur %(8901183002)# Jaipur #𝕔ALL #𝕘IRLS
𝕔ALL #𝕘IRLS Service in Jaipur %(8901183002)#  Jaipur #𝕔ALL #𝕘IRLS𝕔ALL #𝕘IRLS Service in Jaipur %(8901183002)#  Jaipur #𝕔ALL #𝕘IRLS
𝕔ALL #𝕘IRLS Service in Jaipur %(8901183002)# Jaipur #𝕔ALL #𝕘IRLS
 
PT MANAGEMENT OF URINARY INCONTINENCE.pptx
PT MANAGEMENT OF URINARY INCONTINENCE.pptxPT MANAGEMENT OF URINARY INCONTINENCE.pptx
PT MANAGEMENT OF URINARY INCONTINENCE.pptx
 
Antiplatelets in IHD, Dose Duration, DAPT vs SAPT
Antiplatelets in IHD, Dose Duration, DAPT vs SAPTAntiplatelets in IHD, Dose Duration, DAPT vs SAPT
Antiplatelets in IHD, Dose Duration, DAPT vs SAPT
 
End Feel -joint end feel - Normal and Abnormal end feel
End Feel -joint end feel - Normal and Abnormal end feelEnd Feel -joint end feel - Normal and Abnormal end feel
End Feel -joint end feel - Normal and Abnormal end feel
 
Relationship between vascular system disfunction, neurofluid flow and Alzheim...
Relationship between vascular system disfunction, neurofluid flow and Alzheim...Relationship between vascular system disfunction, neurofluid flow and Alzheim...
Relationship between vascular system disfunction, neurofluid flow and Alzheim...
 
Gauri Gawande(9) Constipation Final.pptx
Gauri Gawande(9) Constipation Final.pptxGauri Gawande(9) Constipation Final.pptx
Gauri Gawande(9) Constipation Final.pptx
 
Vaccines: A Powerful and Cost-Effective Tool Protecting Americans Against Dis...
Vaccines: A Powerful and Cost-Effective Tool Protecting Americans Against Dis...Vaccines: A Powerful and Cost-Effective Tool Protecting Americans Against Dis...
Vaccines: A Powerful and Cost-Effective Tool Protecting Americans Against Dis...
 
In-service education (Nursing Mangement)
In-service education (Nursing Mangement)In-service education (Nursing Mangement)
In-service education (Nursing Mangement)
 
Retinal consideration in cataract surgery
Retinal consideration in cataract surgeryRetinal consideration in cataract surgery
Retinal consideration in cataract surgery
 
Prix Galien International 2024 Forum Program
Prix Galien International 2024 Forum ProgramPrix Galien International 2024 Forum Program
Prix Galien International 2024 Forum Program
 
Hemodialysis: Chapter 2, Extracorporeal Blood Circuit - Dr.Gawad
Hemodialysis: Chapter 2, Extracorporeal Blood Circuit - Dr.GawadHemodialysis: Chapter 2, Extracorporeal Blood Circuit - Dr.Gawad
Hemodialysis: Chapter 2, Extracorporeal Blood Circuit - Dr.Gawad
 
Effects of vaping e-cigarettes on arterial health
Effects of vaping e-cigarettes on arterial healthEffects of vaping e-cigarettes on arterial health
Effects of vaping e-cigarettes on arterial health
 
Introducing VarSeq Dx as a Medical Device in the European Union
Introducing VarSeq Dx as a Medical Device in the European UnionIntroducing VarSeq Dx as a Medical Device in the European Union
Introducing VarSeq Dx as a Medical Device in the European Union
 
Cardiovascular Physiology - Regulation of Cardiac Pumping
Cardiovascular Physiology - Regulation of Cardiac PumpingCardiovascular Physiology - Regulation of Cardiac Pumping
Cardiovascular Physiology - Regulation of Cardiac Pumping
 
Final CAPNOCYTOPHAGA INFECTION by Gauri Gawande.pptx
Final CAPNOCYTOPHAGA INFECTION by Gauri Gawande.pptxFinal CAPNOCYTOPHAGA INFECTION by Gauri Gawande.pptx
Final CAPNOCYTOPHAGA INFECTION by Gauri Gawande.pptx
 
Arterial health throughout cancer treatment and exercise rehabilitation in wo...
Arterial health throughout cancer treatment and exercise rehabilitation in wo...Arterial health throughout cancer treatment and exercise rehabilitation in wo...
Arterial health throughout cancer treatment and exercise rehabilitation in wo...
 

TEST BANK For Guyton and Hall Textbook of Medical Physiology, 14th Edition by John E. Hall; Michael E. Hall, Verified Chapters 1 - 86, Complete Newest Version.pdf

  • 1. Test Banks For Guyton and Hall Textbook of Medical Physiology 14th Edition by John E. Hall; Michael E. Hall, Chapters 1 - 86
  • 2. Guyton and Hall Textbook of Medical Physiology 14th Edition Test Bank by John E. Hall, Michael E. Hall Table of Contents Unit I. Introduction to Physiology: The Cell and General Physiology Chapter 1. Functional Organization of the Human Body and Control of the “Internal Environment” Chapter 2. The Cell and Its Functions Chapter 3. Genetic Control of Protein Synthesis, Cell Function, and Cell Reproduction Unit II. Membrane Physiology, Nerve, and Muscle Chapter 4. Transport of Substances Through Cell Membranes Chapter 5. Membrane Potentials and Action Potentials Chapter 6. Contraction of Skeletal Muscle Chapter 7. Excitation of Skeletal Muscle: Neuromuscular Transmission and Excitation- Contraction Coupling Chapter 8. Excitation and Contraction of Smooth Muscle Unit III. The Heart Chapter 9. Cardiac Muscle; The Heart as a Pump and Function of the Heart Valves Chapter 10. Rhythmical Excitation of the Heart Chapter 11. Fundamentals of Electrocardiography Chapter 12. Electrocardiographic Interpretation of Cardiac Muscle and Coronary Blood Flow Abnormalities: Vectorial Analysis Chapter 13. Cardiac Arrhythmias and Their Electrocardiographic Interpretation Unit IV. The Circulation Chapter 14. Overview of the Circulation: Pressure, Flow, and Resistance Chapter 15. Vascular Distensibility and Functions of the Arterial and Venous Systems Chapter 16. The Microcirculation and Lymphatic System: Capillary Fluid Exchange, Interstitial Fluid, and Lymph Flow Chapter 17. Local and Humoral Control of Tissue Blood Flow Chapter 18. Nervous Regulation of the Circulation and Rapid Control of Arterial Pressure Chapter 19. Role of the Kidneys in Long-Term Control of Arterial Pressure and in Hypertension: The Integrated System for Arterial Pressure Regulation Chapter 20. Cardiac Output, Venous Return, and Their Regulation Chapter 21. Muscle Blood Flow and Cardiac Output During Exercise; the Coronary Circulation and Ischemic Heart Disease Chapter 22. Cardiac Failure Chapter 23. Heart Valves and Heart Sounds; Valvular and Congenital Heart Defects Chapter 24. Circulatory Shock and Its Treatment Unit V. The Body Fluids and Kidneys Chapter 25. Regulation of Body Fluid Compartments: Extracellular and Intracellular Fluids; Edema Chapter 26. The Urinary System: Functional Anatomy and Urine Formation by the Kidneys Chapter 27. Glomerular Filtration, Renal Blood Flow, and Their Control Chapter 28. Renal Tubular Reabsorption and Secretion
  • 3. Chapter 29. Urine Concentration and Dilution; Regulation of Extracellular Fluid Osmolarity and Sodium Concentration Chapter 30. Renal Regulation of Potassium, Calcium, Phosphate, and Magnesium; Integration of Renal Mechanisms for Control of Blood Volume and Extracellular Fluid Volume Chapter 31. Acid–Base Regulation Chapter 32. Diuretics and Kidney Diseases Unit VI. Blood Cells, Immunity, and Blood Coagulation Chapter 33. Red Blood Cells, Anemia, and Polycythemia Chapter 34. Resistance of the Body to Infection: I. Leukocytes, Granulocytes, the Monocyte- Macrophage System, and Inflammation Chapter 35. Resistance of the Body to Infection: II. Immunity and Allergy Chapter 36. Blood Types; Transfusion; and Tissue and Organ Transplantation Chapter 37. Hemostasis and Blood Coagulation Unit VII. Respiration Chapter 38. Pulmonary Ventilation Chapter 39. Pulmonary Circulation, Pulmonary Edema, and Pleural Fluid Chapter 40. Principles of Gas Exchange; Diffusion of Oxygen and Carbon Dioxide Through the Respiratory Membrane Chapter 41. Transport of Oxygen and Carbon Dioxide in Blood and Tissue Fluids Chapter 42. Regulation of Respiration Chapter 43. Respiratory Insufficiency—Pathophysiology, Diagnosis, Oxygen Therapy Unit VIII. Aviation, Space, and Deep-Sea Diving Physiology Chapter 44. Aviation, High Altitude, and Space Physiology Chapter 45. Physiology of Deep-Sea Diving and Other Hyperbaric Conditions Unit IX. The Nervous System: A. General Principles and Sensory Physiology Chapter 46. Organization of the Nervous System, Basic Functions of Synapses, and Neurotransmitters Chapter 47. Sensory Receptors, Neuronal Circuits for Processing Information Chapter 48. Somatic Sensations: I. General Organization, Tactile and Position Senses Chapter 49. Somatic Sensations: II. Pain, Headache, and Thermal Sensations Unit X. The Nervous System: B. The Special Senses Chapter 50. The Eye: I. Optics of Vision Chapter 51. The Eye: II. Receptor and Neural Function of the Retina Chapter 52. The Eye: III. Central Neurophysiology of Vision Chapter 53. The Sense of Hearing Chapter 54. The Chemical Senses—Taste and Smell Unit XI. The Nervous System: C. Motor and Integrative Neurophysiology Chapter 55. Spinal Cord Motor Functions; the Cord Reflexes Chapter 56. Cortical and Brain Stem Control of Motor Function Chapter 57. Cerebellum and Basal Ganglia Contributions to Overall Motor Control Chapter 58. Cerebral Cortex, Intellectual Functions of the Brain, Learning, and Memory
  • 4. Chapter 59. The Limbic System and the Hypothalamus—Behavioral and Motivational Mechanisms of the Brain Chapter 60. States of Brain Activity—Sleep, Brain Waves, Epilepsy, Psychoses, and Dementia Chapter 61. The Autonomic Nervous System and the Adrenal Medulla Chapter 62. Cerebral Blood Flow, Cerebrospinal Fluid, and Brain Metabolism Unit XII. Gastrointestinal Physiology Chapter 63. General Principles of Gastrointestinal Function—Motility, Nervous Control, and Blood Circulation Chapter 64. Propulsion and Mixing of Food in the Alimentary Tract Chapter 65. Secretory Functions of the Alimentary Tract Chapter 66. Digestion and Absorption in the Gastrointestinal Tract Chapter 67. Physiology of Gastrointestinal Disorders Unit XIII. Metabolism and Temperature Regulation Chapter 68. Metabolism of Carbohydrates and Formation of Adenosine Triphosphate Chapter 69. Lipid Metabolism Chapter 70. Protein Metabolism Chapter 71. The Liver Chapter 72. Dietary Balances; Regulation of Feeding; Obesity and Starvation; Vitamins and Minerals Chapter 73. Energetics and Metabolic Rate Chapter 74. Body Temperature Regulation and Fever Unit XIV. Endocrinology and Reproduction Chapter 75. Introduction to Endocrinology Chapter 76. Pituitary Hormones and Their Control by the Hypothalamus Chapter 77. Thyroid Metabolic Hormones Chapter 78. Adrenocortical Hormones Chapter 79. Insulin, Glucagon, and Diabetes Mellitus Chapter 80. Parathyroid Hormone, Calcitonin, Calcium and Phosphate Metabolism, Vitamin D, Bone, and Teeth Chapter 81. Reproductive and Hormonal Functions of the Male (and Function of the Pineal Gland) Chapter 82. Female Physiology Before Pregnancy and Female Hormones Chapter 83. Pregnancy and Lactation Chapter 84. Fetal and Neonatal Physiology Unit XV. Sports Physiology Chapter 85. Sports Physiology
  • 5. Chapter 1. Functional Organization of the Human Body and Control of the “Internal Environment” Test Bank 1. The most abundant type of cell in the human body is which of the following? A. Neuron B. Epithelial cell C. Red blood cell D. White blood cell E. Vascular smooth muscle cell F. Skeletal muscle cell ANSWER: C 2. The most abundant substance in the human body and the approximate percentage ofthat substance in the body is which of the following? A. Protein, 30% B. Protein, 60% C. Water, 30% D. Water, 60% E. Carbohydrate, 30% F. Carbohydrate, 60% ANSWER: D 3. A large volume of blood is transfused to a person whose baroreceptor blood pressure control system is not functioning and arterial blood pressure rises from the normal level of 100 to 160 mm Hg. If the same volume of blood is infused into the same person when the baroreceptor system is functioning and this time the arterial pressure increases from the normal level from 100 mm Hg up to 120 mm Hg, calculate the gain of the baroreceptor system in this person. A. -3 B. -2 C. -1 D. 0
  • 6. E. +1 F. +2 G. +3 ANSWER: B 4. Which of the following substances has the highest extracellular fluid to intracellular fluid concentration ratio for most mammalian cells? A. Sodium ions B. Potassium ions C. Carbon dioxide D. Glucose E. Protein ANSWER: A 5. Exchange of substances between the cardiovascular system and the interstitial fluid occurs mainly in which of the following? A. Arteries B. Arterioles C. Capillaries D. Venules E. Veins ANSWER: C 6. Which of the following is the approximate distance from the capillaries to most cellsof the body? A. Less than 50 angstroms B. Less than 50 microns C. Less than 50 millimeters D. Less than 100 angstroms E. Less than 100 microns F. Less than 100 millimeters ANSWER: A 7. When a person is at rest, how much time is required for the blood in the circulation to traverse the entire circulatory circuit? A. 1 second B. 1 minute C. 3 minutes D. 4 minutes E. 5 minutes ANSWER: B
  • 7. 8. feedback is often referred to as a "vicious cycle" because it leads to instability and sometimes death. A. Postitive, progressive B. Positive, diminished C. Negative, progressive D. Negative, diminished E. Adaptive, progressive ANSWER: A 9. Which of the following is an example of positive feedback in the body? A. Clotting of blood B. Return of blood pressure toward normal after a hemorrhage C. Increased respiration rate caused by accumulation of carbon dioxide in the blood D. Decreased sympathetic nervous system activity that occurs in response to increased blood pressure ANSWER: A Chapter 2. The Cell and Its Functions Test Bank Refer to the following list to answer questions 1-3: A. Nucleolus B. Nucleus C. Agranular endoplasmic reticulum D. Granular endoplasmic reticulum E. Golgi apparatus F. Endosomes G. Peroxisomes H. Lysosomes I. Cytosol Identify the cellular location for each of the following steps involved in the synthesis and packaging of a secreted protein. Initiation of translation. ANSWER: I
  • 8. Protein sorting and packaging. ANSWER: E Gene transcription. ANSWER: B 4. Which of the following is true for both pinocytosis and phagocytosis? A. Involves the recruitment of actin filaments B. Occurs spontaneously and non-selectively C. Permits the uptake of bacterium into the cytosol D. Is only observed in macrophages and neutrophils E. Does not require ATP ANSWER: A 5. The cell membrane is LEAST permeable to which of the following? A. Sodium B. Oxygen C. Ethanol D. Carbon Dioxide E. Water ANSWER: A 6. The term “glycocalyx” refers to: A. The negatively charged carbohydrate chains that protrude into the cytosol from glycolipids and integral glycoproteins B. The negatively charged carbohydrate layer on the outer cell surface C. The layer of anions aligned on the cytosolic surface of the plasma membrane D. The large glycogen stores found in “fast” muscles E. A mechanism of cell-cell attachment ANSWER: B 7. Proteins are sorted for their delivery to lysosomes, secretory vesicles and the plasma membrane in the: A. Golgi apparatus B. smooth endoplasmic reticulum C. nucleus D. endocytotic vesicle ANSWER: A 8. Ubiquinone, an electron acceptor in the electron transport chain (oxidative phosphorylation), is found in the:
  • 9. A. Inner mitochondrial membrane B. Mitochondrial matrix C. Outer mitochondrial membrane D. Nucleus ANSWER: A 9. The citric acid cycle or Kreb’s cycle, takes place in the: A. Mitochondrial matrix B. Inner mitochondrial membrane C. Outer mitochondrial membrane D. Inner mitochondrial space ANSWER: A 10. Which of the following processes is NOT ATP-dependent? A. Ciliary movement B. Positive chemotaxis C. Movement of carbon dioxide across a lipid bilayer D. Endocytosis E. Smooth muscle contraction ANSWER: C 11. This cytoskeletal element plays a role in certain forms of cell movement and is an essential component of the mitotic spindle: A. Phospholipids B. Glycocalyx C. F-actin D. Microtubules E. Clathrin ANSWER: D 12. Lipid synthesis occurs in the: A. Trans-Golgi network B. Granular or “rough” endoplasmic reticulum C. Agranular or “smooth” endoplasmic reticulum D. Nucleus E. Lysosome ANSWER: C 13. This cytoskeletal element plays a role in certain forms of cell movement and is an essential component of the mitotic spindle: A. Phospholipids B. Glycocalyx
  • 10. C. F-actin D. Microtubules E. Clathrin ANSWER: D 14. The abnormal cleavage of mannose residues during the post-translational processing of glycoproteins has been shown to result in the development of a lupus-like autoimmune disease in mice. The abnormal cleavage is due to a mutation of the enzyme -mannosidase II. Based on your understanding of the processing of membrane proteins, you would predict this enzyme to be localized to the: A. Nucleus B. Cytosol C. Golgi apparatus D. Lysosomes E. Peroxisomes ANSWER: C 15. The observation that abnormal cleavage of mannose residues from glycoproteins causes an autoimmune disease in mice is most consistent with the role of which of the following structures in the normal immune response? A. Cytoskeleton B. Glycocalyx C. Peroxisomes D. Lysosomes E. Microtubules ANSWER: B 16. A pure phospholipid bilayer is most permeable to: A. Sodium B. Calcium C. Chloride D. Water E. Oxygen ANSWER: E Chapter 3. Genetic Control of Protein Synthesis, cell function, and cell reproduction
  • 11. Test Bank 1. Facioscapulohumeral muscular dystrophy (FSHD) is characterized by the deletion of a DNA sequence on chromosome 4q35. This deletion correlates with both the inability of a specific protein complex to bind to the DNA and an overexpression of the genes upstream of the deletion. The sequence deleted in FSHD most likely functions normally as: A. An activator protein B. A repressor protein C. An activator element D. A repressor element E. A promoter sequence ANSWER: D 2. In comparing two cell types from the same person (e.g. a neuron and an epithelial cell) the variation in their proteomes, or the proteins expressed by each cell type, reflects: A. Differences in the DNA contained in the nucleus of each cell B. Variation in the numbers of copies of specific genes in their respective genomes C. Cell-dependent expression and/or repression of specific genes D. Differences in the number of chromosomes in each cell E. The loss of genes from each genome over time ANSWER: C 3. Which of the following correctly describes the sequence of events that occur during the synthesis and packaging of a secreted protein? A. The gene is transcribed in the cytosol; mRNA is translated by ribosomes bound to “rough” endoplasmic reticulum; the protein is packaged for secretion in the trans- Golgi network B. The gene is transcribed in the nucleus; RNA polymerase binds to the start codon; the protein is packaged for secretion in the trans-Golgi network C. Translation is initiated in the cytosol; the protein is carried in membrane- bound vesicles to the cis-Golgi apparatus; the protein is packaged for secretion in the trans- Golgi network D. Translation is initiated in the cytosol; new polypeptide is co-transported with Na + across the outer ER membrane; the protein is glycosylated in the Golgi apparatus E. Translation is initiated by small ribosomal subunits anchored to the ER membrane; protein is glycosylated in the Golgi apparatus; the protein is packaged into lysosomes ANSWER: C 4. Which of the following does NOT play a direct role in the process of transcription? A. Helicase B. RNA polymerase
  • 12. C. A chain terminating sequence D. “Activated” RNA molecules E. A promoter sequence ANSWER: A 5. “Redundancy” or “degeneration” of the genetic code occurs during which of the following steps of protein synthesis? A. DNA replication B. Transcription C. Post-transcriptional modification D. Translation E. Protein glycosylation ANSWER: B 6. Which of the following bases is NOT present in RNA? A. Cytosine B. Thymine C. Adenine D. Guanine ANSWER: B 7. The process of translation takes place: A. In the cytosol and on the surface of the “rough” endoplasmic reticulum B. In the nucleus and on the surface of the “rough” endoplasmic reticulum C. In the cytosol and the trans-Golgi network (TGN) D. In the nucleus and on the outer mitochondrial membrane ANSWER: A 8. Which of the following statements about translation is NOT true? A. Multiple ribosomes can simultaneously translate a single mRNA molecule B. Each codon codes for one amino acid C. One mRNA molecule can code for multiple proteins, depending on which start codon is recognized by the small ribosomal subunit D. Translation is terminated when the release factor binds to the stop codon ANSWER: C 9. The following statements accurately describe the process of DNA replication EXCEPT: A. The entire genome is replicated only once per cell cycle B. It occurs during the M phase of the cell cycle C. Nucleotides are incorporated into the growing DNA strand at the 3’ end
  • 13. D. DNA “proofreading” is performed by DNA polymerase ANSWER: C 10. The mechanism by which allolactose regulates the transcription of the beta- galactosidase gene is best described as: A. De-repression B. Repression C. Activation D. Negative feedback E. Positive feedback ANSWER: A 11. Which of the following does NOT occur during the process of mitosis? A. Replication of the genome B. Condensation of the chromosomes C. Fragmentation of the nuclear envelope D. Alignment of the chromatids along the equatorial plate E. Separation of the chromatids into two sets of 46 “daughter” chromosomes ANSWER: A 12. “Redundancy” or “degeneration” of the genetic code occurs during which of the following steps of protein synthesis: A. DNA replication B. Transcription C. Post-transcriptional modification D. Translation E. Protein glycosylation ANSWER: B 13. The appearance of which of the following distinguishes eukaryotic cells from lower units of life like bacteria and viruses? A. DNA B. RNA C. Membranes D. Protein E. Nucleus ANSWER: E
  • 14. Chapter 4. Transport of Substances Through Cell Membranes Test Bank 1. An artificial membrane is created consisting of a lipid bilayer without protein molecules in the membrane. The lipid composition of the membrane is essentially the same as that of a normal, biological membrane. Which of the following substances permeates the membrane more readily than water molecules? A. Carbon Dioxide B. Glucose C. Glycerol D. Sodium E. Urea ANSWER: A 2. A cell is equilibrated in an aqueous solution of 300 mOsm/L sodium chloride. Which of the following best describes what will happen to cell volume when the cell is placed in an aqueous solution of 300 mOsm/L calcium chloride? A. Decrease B. Decrease and then increase C. Increase D. Increase and then decrease E. No change ANSWER: E 3. The intracellular calcium ion concentration of ventricular muscle cells averages 10 -4 mmol/L during diastole. The calcium ion concentration in transverse tubules (T-tubules) averages 2.5 mmol/L at rest. A protein transporter on the membrane of the T-tubule exchanges sodium for calcium. The transporter uses the transmembrane sodium gradient to fuel the exchange. Which of the following transport mechanisms best describes this type of transporter? A. Facilitated diffusion B. Primary active transport C. Secondary active co-transport D. Secondary active counter-transport E. Simple diffusion ANSWER: D
  • 15. 4. Human red blood cells (RBCs) and rabbit RBCs are equilibrated in separate solutions of isotonic saline (300 mOsm/L NaCl). The human RBCs are then placed in a solution of 300 mOsm/L glycerol, which causes them to swell and burst. However, rabbit RBCs placed in 300 mOsm/L glycerol neither swell nor shrink. Based on this information, which of the following can be concluded about a 300 mOsm/L solution of glycerol for the different cell types? Human RBCs Rabbit RBCs A. Hypertonic and hyperosmotic Hypotonic and hypoosmotic B. Hypotonic and hypoosmotic Hypertonic and hyperosmotic C. Hypotonic and isoosmotic Isotonic and isoosmotic D. Isotonic and hypoosmotic Isotonic and hyperosmotic E. Isotonic and isoosmotic Hypotonic and isoosmotic F. Isotonic and hyperosmotic Isotonic and isoosmotic ANSWER: C 5. The molarity of a 2% solution of NaCl is 340 mmol/L. The molecular weight of NaCl is 58.5. What is the osmolarity of a 2% solution of NaCl (in mOsm/L)? A. 170 B. 340 C. 510 D. 680 ANSWER : D 6. Secondary active transport typically moves which of the following substances against a concentration gradient? Glucose A. No Amino acids No Sodium ions No B. No No Yes C. Yes No Yes D. Yes E. Yes Yes Yes No Yes ANSWER: D 7. Which of the following transport mechanisms can move sodium ions across a cell membrane? Primary active transport No No No Yes Secondary active transport No Yes Simple diffusio n Yes No Yes Yes Yes No Yes Yes Yes
  • 16. ANSWER: E 8. The diagram illustrates possible changes in red blood cell volume resulting from a change in extracellular fluid composition for a cell equilibrated in a 150 mmol/L solution of sodium chloride (NaCl) at time zero. Which curve best illustrates the volume change caused by immersion of the cell in an aqueous solution of 300 mOsm/L calcium chloride (CaCl2)? ANSWER: C 9. The diagram illustrates possible changes in red blood cell volume resulting from a change in extracellular fluid composition for a cell equilibrated in a 150 mmol/L solution of sodium chloride (NaCl) at time zero. Which curve best illustrates the volume change caused by immersion of the cell in an aqueous solution of 200 mOsm/L NaCl and 200 mOsm/L glycerol? ANSWER: B 10. Which of the following pairs of aqueous solutions will exert equal osmotic pressures across a normal cell membrane after steady-state conditions have been established? A. Solution A 10% albumin Solution B 10% IgG B. C. 100 mmol/L NaCl 300 mOsm/L glucose 200 mmol/L CaCl2 300 mOsm/L urea D. E. 300 mOsm/L glycerol 300 mOsm/L glycerol 300 mOsm/L NaCl 300 mOsm/L urea ANSWER: E 11. Two compartments (X and Y) are separated by a typical biological membrane (i.e., lipid bilayer). The concentrations of a permeant solute (i.e., urea) at time zero are shown. Which of the drawings below represents the volumes of X and Y when the system reaches equilibrium? ANSWER: A 12. The diagram illustrates possible changes in red blood cell volume resulting from a change in extracellular fluid composition for a cell equilibrated in 150 mmol/L NaCl at
  • 17. time zero. Which curve best illustrates the volume caused by immersion of the cell in an aqueous solution of 150 mmol/L CaCl2? ANSWER: E 13. Two compartments (X and Y) are separated by a typical biological membrane (lipid bilayer). The concentrations of a non-permeant molecule (glucose) at time zero are shown. Which of the drawings below represents the volumes of X and Y when the system reaches equilibrium? ANSWER: B 14. The diagram shows a model cell that transports substance X across the cell membrane. The cell is equipped with a Na-K-ATPase pump as shown. Substance X enters the cell by a coupled transport mechanism and exits the cell by carrier-mediated diffusion. Treatment with a substance that inhibits the Na-K- ATPase pump inhibits the transport of X by which of the following mechanisms? A. Decreasing intracellular K + concentration + B. Decreasing intracellular Na + concentration C . Increasing intracellular K concentration D. Increasing intracellular Na + concentration ANSWER: D 15. The diagram shows a bag (with permeability characteristics similar to that of a normal cell) that contains a 100 mM solution of urea at time zero. The bag is placed in a beaker containing 100 mM glucose. Which of the following best describes the tonicity and osmolarity of the glucose solution as well as any changes in bag volume (assume that the bag volume is infinitely small compared to beaker volume)? Osmolarity Tonicity Bag volume A. Hyperosmotic Hypertonic Decreases B. Hyperosmotic Hypotonic Increases C. Hyperosmotic Isotonic No change D. Hypoosmotic Hypotonic Decreases
  • 18. E. Hypoosmotic Isotonic Increases F. Hypoosmotic Hypertonic No change G. Isoosmotic Hypertonic Decreases H. Isoosmotic Hypotonic Increases I. Isoosmotic Isotonic No change ANSWER: G Chapter 5. Membrane Potentials and Action Potentials Test Bank 1. The diagram shows compound action potentials recorded percutaneously from a single site a certain distance from a stimulating electrode indicated by the arrow. Which peak is most likely to represent action potentials from small, non-myelinated motor neurons? ANSWER: D 2. A 42-year-old woman decides to lose weight on a diet prescribed by an anorexic friend. She loses about 30 pounds in 45 days, but her serum potassium level falls to 2.1 mmol/L (normal: 3.5 to 5.0 mmol/L). Which of the following changes is most likely to occur in this young woman? A. Depolarization of the resting membrane potential B. Hyperpolarization of the resting membrane potential C. Increased intracellular potassium concentration D. Potassium equilibrium potential becomes less negative ANSWER: B 3. An 22-year-old man is admitted to the emergency department after an automobile accident. He has not lost a large amount of blood, but he suffers from a severe crush injury to his right leg. He is conscious and exhibits flaccid paralysis. Electrocardiogram
  • 19. Na results show a shortened QT interval and an increase in the P-R interval (i.e., decreased duration of action potential and decreased conduction velocity). Laboratory results indicate that his serum potassium level has increased to a value of 6.5 mEq/L (normal: 3.5 to 5.0 mEq/L). This increase in serum potassium causes which of the following? A. A decreased net driving force for potassium from cells B. Decreased intracellular potassium concentration C. Hyperpolarization of the resting membrane potential D. More negative Nernst potential for potassium E. More negative resting membrane potential ANSWER: A 4. An 88-year-old nursing home resident has diabetes mellitus and is taking diuretics for hypertension. His blood glucose concentration has recently been greater than 350 mg/dL. He is disoriented and has had limited oral intake for about five days. His vital signs include a temperature of 100.5°F, a supine blood pressure and pulse of 150/90 mm Hg and 100 beats per minute, and an upright blood pressure and pulse of 130/88 mm Hg and 118 beats per minute. The findings on examination of the heart are normal, and his axillae + + a re dry. The table shows serum N K a and values before and after the dehydration that has occurred in this patient. The resting membrane potential of a cardiac myocyte was about −70 millivolts before dehydration occurred. What is the resting membrane potentialof a cardiac myocyte after dehydration occurred, assuming no change in membrane conductance or intracellular concentrations of + or K + ions? A. −100 millivolts B. −50 millivolts C. −60 millivolts D. −70 millivolts E. −90 millivolts ANSWER: D 5. A 44-year-old man is recovering from third degree burns over 50% of his body surface area. His serum potassium level is low (2.9 mEq/L). He exhibits postural hypotension and skeletal muscle weakness. If the equilibrium potential for K + in a neuron is −105 mV and the equilibrium potential for Na + is +55 mV, what would be an approximate value for the resting membrane potential for a neuron in this man? A. +20 mV B. +48 mV C. +80 mV D. −25 mV E. −48 mV F. −96 mV ANSWER: F
  • 20. 6. A 23-year-old man receives a crush injury to his right leg in an automobile accident. His serum potassium level is 6.4 mEq/L. The equilibrium potential for K + in a motoneuron is −73 mV, and the equilibrium potential for Na + is +59 mV. Which of the following best describes the resting membrane potential (in mV) for a typical motoneuron in this man? A. +62 B. +90 C. −13 D. −67 E. −84 ANSWER: D 7. A 56-year-old woman is referred to the neurology clinic for evaluation of her long- term problem of a stumbling gait and a tendency to fall. Her visual acuity also seems to change periodically. Neurological findings suggest multiple sclerosis. This diagnosis is further supported by magnetic resonance imaging, which shows of areas of demyelination in the central nervous system. Other tests show the presence of oligoclonal bands in the spinal fluid. The loss of myelin from a nerve fiber has which of the following effects? A. Decreased conduction velocity of action potential or no action potential B. Decreased energy requirement for successful action potentials C. Increased conduction velocity of action potential D. Increased velocity of salutatory conduction ANSWER: A 8. The diagram shows an action potential for a nerve fiber. What is the threshold potential for this neuron? A. −15 mV B. −50 to −60 mV C. −60 to −70 mV D. −70 to −80 mV E. −80 mV ANSWER: C 9. In the diagram shown, Em represents the measured initial membrane potential for a hypothetical cell in vivo. In relation to this membrane potential, the equilibrium potentials
  • 21. of three ions (X, Y, Z) are represented. Pick the path most likely taken by the membrane potential when membrane conductance for ion Y is increased. ANSWER: B 10. In the diagram shown, Pmemb represents the initial membrane potential for a hypothetical cell. In relation to this membrane potential, the equilibrium potentials of three ions (Q, R, S) are represented. Pick the path most likely taken by the membrane potential when membrane conductance for ion Q and ion S are increased simultaneously by 1000-fold. ANSWER: C 11. The diagram shows compound action potentials recorded percutaneously from a single site at a certain distance from a stimulating electrode (S) indicated by the ARROW. What can be concluded from the diagram concerning action potential velocity for the various categories of neurons? A. W = X = Y = Z B. W > X > Y > Z C. W > Y > X D. Y > W > Z E. Z > Y > X > W ANSWER: B 12. Five hypothetical neurons are shown below. Neurons A and B are myelinated, whereas neurons C, D, and E are not myelinated. Which neuron is most likely to have the fastest conduction velocity? A. B. C. D. E. ANSWER: B
  • 22. 13. In the following graph, Em represents the measured initial membrane potential for a hypothetical cell in vivo. In relation to this membrane potential, the equilibrium potentials of three ions (Q, R, S) are represented. During resting conditions, the conductance for Q and S ions is the same, which is 1000 times greater than that of ion R. Pick the path most likely taken by the membrane potential when membrane conductance to ion Q is 2 decreased to 0 mmho/cm . ANSWER: D 14. A 63-year-old man is dehydrated after three days of alcohol abuse. His serum sodium level has increased from a normal value of 140 mEq/L to 147 mEq/L. The resting membrane potential of a typical neuron was −80 mV before dehydration. What is the most likely resting membrane potential of a typical neuron after the dehydration, assuming no change in resting sodium conductance? A. −72 mV B. −74 mV C. −80 mV D. −88 mV E. −98 mV ANSWER: C 15. A 24-year-old soldier suffers a burn injury over 60% of his body surface area, which + l eads to a decrease in the serum K concentration from a normal value of 4 mEq/L to a value of 2.5 mEq/L. Which of the following sets of changes best describe the resting membrane potential and the K + Nernst potential in a cardiac myocyte after the injury? + Resting membrane potential K Nernst potential A. Less negative Less negative B. Less negative More negative C. More negative Less negative D. More negative More negative E. No change No change ANSWER: D
  • 23. Ca 16. The diagram shows an action potential for a nerve fiber. Use the diagram to answer the question. Where on the curve does potassium conductance exceed sodium conductance? A. V and Z B. V, W, and X C. V, X, and Z D. W, X, and Y E. X, Y, and Z ANSWER: A 17. Consider the action potential shown. Which of the following changes in membrane conductances for + and K + best account for the prolonged plateau (assume normal conductances for both ions during resting conditions, i.e., only the changes in conductances are important)? A. B. C. D. ANSWER: A 18. A neuron from an extraterrestrial being has the intracellular and extracellular sodium and potassium concentrations shown in the table (in mM). During resting conditions, the sodium conductance is 100-fold greater compared to the potassium conductance. Which of the following (in mV) best describes the resting membrane potential (Vm) in this alien neuron (assume that Vm is dictated mainly by sodium and potassium conductances and that the Nernst equation applies)? A. +150 B. +55 C. 0 D. −55 E. −150 ANSWER: B
  • 24. 19. The intracellular concentration of ion Z is 100 milliequivalents per liter (mEq/L), and the extracellular concentration is 5 mEq/L. The molecular weight of molecule Z is ~40 daltons. This relatively high intracellular concentration of molecule Z is best explained by which of the following? A. Exocytosis B. Extracellular assembly C. Facilitative diffusion D. Intracellular production E. Primary active transport F. Simple diffusion ANSWER: E 20. Equilibrium potentials for Na, K, and Cl are shown. Assume that a cell is permeable to all three ions and has a resting membrane potential of −85 mV. Which of the following best describes the net movement of the various ions across the cell membrane by way of passive diffusion? Sodium Potassium Chloride A. Inward Inward Inward B. Inward Inward Outwar d C. Inward Outward Inward D. Inward Outward Outwar d E. Outward Inward Inward F. Outward Inward Outwar d G. Outward H. Outward Outward Outward Inward Outwar d ANSWER: D Chapter 6. Contraction of Skeletal Muscle Test Bank 1. A 28-year-old man has been training as a distance runner for the past six years. Compared with a sprinter, which of the following is decreased in the skeletal muscles of this athlete during resting conditions? A. Capillary density B. Glycolytic enzyme content C. Mitochondrial volume density D. Myoglobin content E. Oxidative capacity ANSWER: A
  • 25. 2. A physiology experiment is conducted in which a skeletal muscle twitch is initiated using an electrical stimulator. Which factor contributes to the termination of a typical skeletal muscle twitch? A. Calcium uptake by sarcoplasmic reticulum B. Closure of transverse tubules C. Depletion of calcium stores from sarcoplasmic reticulum D. Exhaustion of muscle ATP stores E. Stimulation of dihydropyridine (DHP) receptors ANSWER: A 3. A 24-year-old medical student goes to the local gym to lift weights. She begins by bench- pressing 100 pounds as a warm-up procedure and then gradually increases the weight. Which of the following occurs as she adds more weight? A. Decreased frequency of motor nerve action potentials B. Decreased velocity of motor nerve action potentials C. Increased frequency of motor nerve action potentials D. Increased velocity of motor nerve action potential E. Involvement of fewer motor units ANSWER: C 4. A 24-year-old woman is admitted to the emergency department after an automobile accident in which lacerations to the left wrist severed a major tendon. The severed ends of the tendon were overlapped by 6 cm to facilitate suturing and reattachment. Which of the following characteristics would be expected after six weeks compared with the pre- injured muscle? Assume that series growth of sarcomeres cannot be completed within six weeks. Passive tension Maximal active tension A. Decrease Decrease B. Decrease Increase C. Increase Increase D. Increase Decrease E. No change No change ANSWER: D 5. During the contraction of a skeletal muscle fiber, the actin and myosin filaments slide past each other. Which of the following represent expected changes in the widths of I- bands and A-bands during the contraction process? I-band width A-band width A. B. C.
  • 26. D. E. ANSWER: D 6. When a person lifts a 25-pound weight, what type or types of muscle contraction are involved? Isometric contraction only Isotonic contraction only Isometric contraction followed by isotonic contraction Isotonic contraction followed by isometric contraction ANSWER: C 7. The length-tension diagram shown was obtained from a skeletal muscle with equal numbers of red and white fibers. Supramaximal tetanic stimuli were used to initiate an isometric contraction at each muscle length studied. The resting length was 20 cm. What is the maximum amount of active tension this muscle is capable of generating at a preload of 100 grams? A. 145 to 155 grams B. 25 to 35 grams C. 55 to 65 grams D. 95 to105 grams E. Cannot be determined ANSWER: C 8. The diagram shows the relationship between muscle tension and sarcomere length for muscle. Which point on the curve represents tension development at a normal resting length for skeletal muscle? ANSWER: C 9. The diagram shows the relationship between muscle tension and sarcomere length for muscle. Which point on the curve represents tension development at a normal resting length for cardiac muscle? ANSWER: B
  • 27. 10. A 29-year-old woman has been training as a distance runner for the past five years. Which set of changes best describes the skeletal muscles of this woman compared with a sprinter? Capillary density Oxidative capacity Myoglobin content Fiber diameter A. B. Decrease d Decrease d Decreased Decreased Decreased Increased Decrease d Increased C. Increased Increased Decreased Decrease d D. E. Increased Increased Increased Increased Increased Increased Decrease d Increased ANSWER: D 11. The length-tension diagram shown on the right was obtained from a skeletal muscle with equal numbers of red and white fibers. Supramaximal tetanic stimuli were used to initiate an isometric contraction at each muscle length studied. What are the values in grams of preload, active tension, and total tension when the muscle length is 60 cm? A. Preloa d 100 Active tension 200 Total tension 300 B. 100 300 200 C. 150 250 300 D. 200 100 300 E. 200 200 300 ANSWER: A 12. A 45-year-old man goes to the local gym to lift weights. He begins by bench- pressing 130 pounds as a warm-up procedure and then gradually increases the weight. Which of the following sets of changes occur as he adds more weight? A. Activation of motor units Decreased Frequency of motor nerve action potentials Decreased B. C. Decreased Decreased Increased No change D. E. F. Increased Increase d Increase d Decreased Increased No change ANSWER: E
  • 28. 13. The skeletal muscle sarcomere normally has a narrow range of lengths in vivo, but sarcomere length can vary greatly in vitro under non-physiological conditions. The diagram shows sarcomeres at various lengths. At which length can the sarcomere exert its maximum force during an isometric contraction? A. 1.6 µm B. 2.1 µm C. 2.8 µm D. 3.0 µm E. 3.5 µm ANSWER: B 14. The diagram shows the relationship between contraction velocity and force. Which of the following muscles is most likely to correspond to muscle number 1 shown on the diagram (assume that all muscles shown are at their normal resting lengths). A. B. C. D. E. ANSW ER: E
  • 29. 15. The diagram shows the relationship between contraction velocity and force. Which of the following muscles is most likely to correspond to muscle number 2 shown on the diagram (assume that all muscles shown are at their normal resting lengths)? A. B. C. D. E. 16. The diagram shows force-velocity relationships for three different skeletal muscles. Which of the following curves most likely correspond to glycolytic, oxidative, and mixed muscle types? Glycolytic Oxidative Mixed A. X Y Z B. X Z Y C. Y X Z D. Y Z X E. Z F. Z X Y Y X ANSWER: B 17. The dark color of red skeletal muscle (slow twitch muscle) can be attributed to a relative abundance of which of the following compared to white skeletal muscle (fast twitch muscle)? A. Blood capillaries B. Mitochondria C. Myoglobin D. Myosin ATPase E. Oxidative enzymes ANSWER: C Chapter 7. Excitation of Skeletal Muscle: Neuromuscular Transmission and ANSW ER: D
  • 30. Excitation-Contraction Coupling Test Bank 1. A 45-year-old man has botulinum toxin (Botox®) injections in the frontalis muscles. Which of the following sets of changes are most likely to occur in the treated muscles with repeated injections over a period of ten years? Fiber diameter Fiber number Contraction velocity A. decreased decreased decreased B. decreased decreased increased C. decreased increased increased D. increased decreased decreased E. increased F. increased decreased increased increased increased ANSWER: A 2. Which of the following represents the correct temporal order of events for skeletal muscle? First A. End plate potential Second Muscle action potential Third Nerve action potential B. End plate potential Nerve action potential Muscle action potential C. Muscle action potential End plate potential Nerve action potential D. Muscle action potential Nerve action potential End plate potential E. Nerve action potential F. Nerve action potential End plate potential Muscle action potential Muscle action potential End plate potential ANSWER: E 3. Myasthenia gravis is an autoimmune disease in which antibodies damage which of the following? Acetylcholine Ligand-gated channels on postsynaptic membrane Ligand-gated channels on presynaptic membrane Voltage-gated channels on postsynaptic membrane Voltage- gated channels on presynaptic membrane ANSWER: B 4. Lambert-Eaton myasthenic syndrome is an autoimmune disease in which antibodies damage which of the following? A. Acetylcholine B. Ligand-gated channels on postsynaptic membrane C. Ligand-gated channels on presynaptic membrane
  • 31. D. Voltage-gated channels on postsynaptic membrane E. Voltage-gated channels on presynaptic membrane ANSWER: E 5. A 59-year-old college professor visits his physician because of muscle fatigue that worsens later in the day. The physician notices that the man has droopy eyelids and seems to have an overall weakness in his muscles. After the intravenous administration of Tensilon®, which blocks degradation of acetylcholine, the droopy eyelids and symptoms of muscle weakness appear to subside. Which of the following best describes the patient’s condition? A. Encephalitis B. Fibromyalgia C. Flaccid paralysis D. Myasthenia gravis E. Poliomyelitis ANSWER: D 6. A 3-year-old child is admitted to the emergency department of University Hospital. The child exhibits extreme salivation, lacrimation, tremors, and tachycardia. The slight erythema and mild edema found on the child's hand suggest a spider bite. The mother acknowledges that the child has a fascination with spiders and has seen black widow spiders in the yard. The neurotoxic venom of the black widow spider (i.e., latrotoxin or latrophilin) can increase the flux of calcium ions into the presynaptic terminal. What is the likely outcome of a black widow spider bite? A. Decreased acetylcholine release from nerve terminals B. Hyperpolarization of postsynaptic membranes C. Hyperpolarization of presynaptic membranes D. Increased acetylcholine release from nerve terminals E. Inhibition of acetylcholinesterase ANSWER: D 7. An apparently healthy 12-year-old boy dies during a minor surgical procedure while under general anesthesia. An uncle had also died during a surgical procedure. Results of a halothane contracture test indicate that the child had malignant hyperthermia (MH). The MH crisis is associated with which of the following? A. Decreased anaerobic metabolism B. Decreased CO2 production by muscles C. Decreased lactic acid production by muscles D. Defective ryanodine receptors E. Decreased usage of ATP in muscles ANSWER: D
  • 32. 8. Skeletal muscle and cardiac muscle contract when calcium binds to which structure or molecule? Active sites Calmodulin Myosin head Tropomyosi n Troponin- C ANSWER: E 9. Which of the following represents the correct temporal order of events for skeletal muscle? First Second Third Muscle action potential Muscle contraction Nerve action potential Muscle action potential Nerve action potential Muscle contraction Muscle contraction Muscle action potential Nerve action potential Muscle contraction Nerve action potential Muscle action potential Nerve action potential Muscle action potential Muscle contraction Nerve action potential Muscle contraction Muscle action potential ANSWER: E 10. Binding of calcium to which structure or molecule initiates a contraction in smooth muscle, skeletal muscle, and cardiac muscle? Smooth muscle Skeletal muscle Cardiac muscle A. Active sites Calmodulin Troponin B. Active sites Troponin Calmodulin C. Calmodulin Tropomyosin Active sites D. Calmodulin E. Myosin heads F. Myosin heads Troponin Tropomyosi n Troponin Troponin Troponin Tropomyosi n ANSWER: D 11. A 23-year-old man loses consciousness while driving, hits a tree, and dies within 15 minutes. The police find his stiff body five hours later. Rigor mortis is caused by a decrease in which of the following? A. Acetylcholine B. Actin-myosin cross-bridges C. Myoplasmic calcium levels D. Interstitial lactate levels E. Muscle ATP levels ANSWER: E 12. A father and his 10-year-old daughter died suddenly of suffocation after ingesting smoked whitefish chubs from the Great Lakes. Three other family
  • 34. complaining of dry mouths despite drinking copious amounts of fluids. Their upper eyelids droop. Their vision is clear, but their pupils are wide and do not narrow when a light is flashed. Two of them require artificial respiration. The suspect is botulism (Clostridium botulinum). The surviving patients begin improving soon after receiving the only treatment available, E antitoxin. The deadly effect of botulinum toxin results from which of the following? A. Decreased release of acetylcholine B. Decreased release of norepinephrine C. Depletion of muscle calcium stores D. Skeletal muscle tetany E. Stimulation of ryanodine receptors ANSWER: A 13. Which of the following best describes the dynamics of calcium release and calcium uptake during the development of frequency-induced muscle tetany? A. Release from longitudinal tubules exceeds uptake by terminal cisternae B. Release from longitudinal tubules exceeds uptake by T-tubules C. Release from terminal cisternae exceeds uptake by longitudinal tubules D. Release from terminal cisternae exceeds uptake by T-tubules E. Release from T-tubules exceeds uptake by longitudinal tubules F. Release from T-tubules exceeds uptake by terminal cisternae ANSWER: C 14. Which of the following best describes the correct temporal order of events for the opening of nicotinic acetylcholine (nACh) channels on the muscle membrane, the opening of voltage-gated sodium channels on the muscle membrane, and initiation of presynaptic action potentials (AP)? First A. nACh channels Second Presynaptic AP Third Voltage gated Na channels B. nACh channels Voltage gated Na channels Presynaptic AP C. Presynaptic AP nACh channels Voltage gated Na channels D. Presynaptic AP Voltage gated Na channels nACh channels E. Voltage gated Na channels F. Voltage gated Na channels nACh channels nACh channels Presynaptic AP Presynaptic AP ANSWER: C Chapter 8. Excitation and Contraction of Smooth Muscle Test Bank
  • 35. 1. The contraction of smooth muscle can be attenuated even when intracellular calcium levels are high. This unique ability of smooth muscle can be attributed to variations in the activity of which of the following? A. Actin B. Adenosine triphosphate (ATP) C. Calcium-calmodulin complex D. Calmodulin E. Myosin light chain phosphatase ANSWER: E 2. Multiple elements can regulate the contraction of smooth muscle, including endocrine, neurocrine, and paracrine factors. This level of diversity in the control of smooth muscle contraction can be attributed to a diversity of which of the following? A. Actin-myosin arrangement B. Calcium sensitivities C. Cell-surface receptors D. Motor end plates E. Subneural cleft architecture ANSWER: C 3. The effect of autonomic nerve stimulation on smooth muscle contractile behavior depends on the types of receptors on the smooth muscle fibers. Which of the following best describes the effects of acetylcholine and norepinephrine on various types of smooth muscle? Acetylcholine Norepinephrine A. Contraction only Relaxation or contraction B. Contraction only Relaxation only C. Relaxation only Contraction only D. Relaxation only Relaxation only E. Relaxation or contraction Contraction only F. Relaxation or contraction Relaxation or contraction ANSWER: F 4. Unlike skeletal muscle, the contraction of smooth muscle requires which of the following? A. Activation of ryanodine receptors B. Phosphorylation of myosin light chains C. The presence of intracellular calcium D. Troponin binding of calcium E. Voltage activation of dihydropyridine receptors ANSWER: B
  • 36. 5. Which of the following factors are important for initiating smooth muscle contractions? Hormones Paracrine factors Autonomic nervous system A. No No No B. No No Yes C. Yes No Yes D. Yes E. Yes Yes Yes No Yes ANSWER: E 6. The sensitivity of the smooth muscle contractile apparatus to calcium can increase. This increase in calcium sensitivity can be attributed to a decrease in the levels of which of the following substances? A. Actin B. Adenosine triphosphate (ATP) C. Calcium-calmodulin complex D. Calmodulin E. Myosin light chain phosphatase ANSWER: E 7. Which of the following is the correct temporal order of events for the smooth muscle? A. Myosin light chain (MLC) dephosphorylation, contraction, relaxation, MLC phosphorylation B. MLC dephosphorylation, contraction, MLC phosphorylation, relaxation C. MLC dephosphorylation, relaxation, MLC phosphorylation, relaxation D. MLC phosphorylation, contraction, MLC dephosphorylation, relaxation E. MLC phosphorylation, contraction, relaxation, MLC dephosphorylation F. MLC phosphorylation, MLC dephosphorylation, contraction, relaxation ANSWER: D Chapter 9. Cardiac Muscle; The Heart as a Pump and Function of the Heart Valves Questions 1-3: A fifty-year-old man has a body weight of 110 kg, resting heart rate is 90 beats/minute, arterial pressure is 120/70 mmHg, arterial hematocrit is 35% and body temperature is normal. His pressure-volume diagram of the left ventricle is shown below. 1. What is his resting cardiac output in ml/minute?
  • 37. A. 900 B. 8,000 C. 9,000 D. 10,000 E. 15,000 ANSWER: C 2. What event occurs at point D in the man’s pressure-volume relationship? A. A-V valves close B. Aortic valve opens C. Aortic valve closes D. A-V valves open E. Third heart sound ANSWER: C 3. What is his ventricular ejection fraction?A. 33% B. 50% C. 60% D. 67% E. 80% ANSWER: D 4. Which of the following will result in spastic contraction of the heart? A. Excess calcium ions in the blood B. Excess potassium ions in the blood C. Excess sodium ions in the blood D. Decreased sympathetic stimulation E. Decreased norepinephrine concentration in the blood ANSWER: A Questions 5-7: A sixty-year-old woman has a body weight of 110 kg, resting cardiac output is 9,000 ml/minute, arterial pressure is 90/50 mmHg, arterial hematocrit is 25% and body temperature is normal. Her pressure-volume diagram of the left ventricle is shown below. 5. What is her resting heart rate in beats/minute? A. 70 B. 80 C. 90 D. 100 E. 110
  • 38. ANSWER: C 6. What event occurs at point A in the woman’s pressure-volume relationship? A. A-V valves close B. Aortic valve opens C. Aortic valve closes D. A-V valves open E. Third heart sound ANSWER: D 7. If the woman is given a 500 ml whole blood transfusion over a 15 minute period, her ventricular preload will increase. One minute after the transfusion is complete, what will the new stroke volume be assuming her heart is normal? A. 70 ml B. 80 ml C. 90 ml D. 100 ml E. 120 ml ANSWER: A Questions 8-9: A fifty-year-old man has a body weight of 110 kg, resting heart rate is 90 beats/minute, arterial pressure is 120/70 mmHg, arterial hematocrit is 35% and body temperature is normal. His pressure-volume diagram of the left ventricle is shown below. 8. When does the third heart sound occur in the ventricular pressure- volume relationship? A. At point D B. Between point A and point B C. Between point B and point C D. Between point C and point D E. Between point D and point A ANSWER: B 9. When does the second heart sound occur in the ventricular pressure- volume relationship? A. At point D B. Between point A and point B C. Between point B and point C D. Between point C and point D E. Between point D and point A ANSWER: A
  • 39. Chapter 10. Rhythmical Excitation of the Heart Test Bank 1. If the sinus node stops discharging, what is the expected heart rate (in beats/minute) if the A-V node takes over as the cardiac pacemaker? A. 15 B. 20 C. 35 D. 50 E. 72 ANSWE R: D 2. If the S-A node discharges at 0.00 seconds, when will the action potential normally arrive at the A-V node? A. 0.03 seconds B. 0.09 seconds C. 0.12 seconds D. 0.16 seconds E. 0.80 seconds ANSWER: A 3. Which of the following conditions at the S-A node will cause a decrease in heart rate? A. Increased sodium permeability B. Decreased acetylcholine levels C. Increased norepinephrine levels D. Increased potassium permeability E. Increased calcium permeability ANSWER: D 4. In which phase of the ventricular muscle action potential is the potassium permeability the highest? A. 0 B. 1 C. 2 D. 3
  • 40. E. 4 ANSWE R: D 5. If the S-A node discharges at 0.00 seconds, when will the action potential normally arrive at the A-V bundle? A. 0.80 seconds B. 0.16 seconds C. 0.12 seconds D. 0.09 seconds E. 0.03 seconds ANSWER: C 6. A patient entered the local emergency room and had an EKG taken. The attending physician stated that the patient had an A-V nodal rhythm. What is the likely heart rate? A. 30/min B. 50/min C. 65/min D. 75/min E. 85/min ANSWER: B 7. Which of the following conditions at the S-A node will cause a decrease in heart rate? A. Increased sodium permeability B. Increased acetylcholine levels C. Increased norepinephrine levels D. Decreased potassium permeability E. Increased calcium permeability ANSWER: B 8. Which of the following conditions at the A-V node will cause a decrease in heart rate? A. Increased sodium permeability B. Decreased acetylcholine levels C. Increased norepinephrine levels D. Increased potassium permeability E. Increased calcium permeability ANSWER: D 9. If the S-A node discharges at 0.00 seconds, when will the action potential normally arrive at the ventricular septum? A. 0.22 seconds B. 0.16 seconds
  • 41. C. 0.12 seconds D. 0.09 seconds E. 0.03 seconds ANSWER: B 10. What is the normal total delay of the cardiac impulse in the A-V node + bundle? A. 0.22 seconds B. 0.18 seconds C. 0.16 seconds D. 0.13 seconds E. 0.09 seconds ANSWER: D 11. Which of the following are caused by acetylcholine? A. Hyperpolarization of the S-A node B. Depolarization of the A-V node C. Decreased permeability of the S-A node to potassium ions D. Increased heart rate E. Increased permeability of the cardiac muscle to calcium ions ANSWER: A 12. If the sinus node stops discharging, what is the expected heart rate (in beats/minute) if the A-V node takes over as the cardiac pacemaker? A. 5 B. 30 C. 50 D. 70 E. 90 ANSWE R: C 13. Sympathetic stimulation of the heart: A. Releases acetylcholine at the sympathetic endings B. Decreases sinus nodal discharge rate C. Decreases excitability of the heart D. Releases norepinephrine at the sympathetic endings E. Decreases cardiac contractility ANSWER: D
  • 42. Chapter 11. The Normal Electrocardiogram Test Bank 1. The Q-T interval is the time that approximates the time of ventricular contraction. The normal Q-T interval is: A. 0.04 seconds B. 0.08 seconds C. 0.16 seconds D. 0.20 seconds E. 0.35 seconds ANSWER: E Questions 2-3: A seventy-year old female had an electrocardiogram performed at her annual checkup, and the following lead II recording was made. 2. What is her heart rate in beats/minute? A. 68 B. 78 C. 84 D. 94 E. 104 ANSWER : A 3. According to Einthoven’s Law, if the QRS voltage in her lead III is 0.5 mV, what is the QRS voltage in Lead I? A. 0.05 mV B. 0.50 mV C. 1.05 mV D. 1.25 mV E. 2.05 mV ANSWER: B 4. The P-R interval is the time between the beginning of atrial depolarization and ventricular depolarization. The normal P-R interval is: A. 0.04 seconds
  • 43. B. 0.08 seconds C. 0.16 seconds D. 0.20 seconds E. 0.35 seconds ANSWER: C 5. When recording lead I on an EKG, the right arm is the negative electrode and the positive electrode is the: A. Left arm B. Left leg C. Right leg D. Left arm + left leg E. Right arm + left leg ANSWER: A Chapter 12. Electrocardiographic Interpretation of Cardiac Muscle and Coronary Blood Flow Abnormalities: Vectorial Analysis Test Bank 1. A ventricular depolarization wave when traveling 210 in the frontal plane will cause a large positive deflection in: A. aVR B. aVL C. Lead II D. Lead III E. aVF ANSWER: A Questions 2-4: A 60-year-old woman had the following EKG recorded at a local emergency room following an automobile accident. Her weight was 70 kg and her aortic blood pressure was 140/80 mmHg. 2. What is the mean electrical axis calculated from standard leads I, II, and III shown in the woman’s EKG?
  • 44. A. −90 B. −50 C. −12 D. +105 E. +170 ANSWER: D 3. What is the heart rate using lead I for the calculation? A. 70 B. 88 C. 100 D. 112 E. 148 ANSWER : A 4. What is her likely diagnosis? A. Mitral valve stenosis B. Left bundle branch block C. Pulmonary valve stenosis D. Right bundle branch block E. Left ventricular hypertrophy ANSWER: D 5. A seventy-five year-old woman came into a local emergency room complaining of chest pain and shortness of breath. Blood samples were taken for cardiac enzyme measurements, and an EKG was quickly taken. Based on the EKG tracing shown below, what is the likely diagnosis? A. Left bundle branch block B. Acute anterior wall infarction in the base of the heart C. Acute anterior wall infarction in the apex of the heart D. Acute posterior infarction in the base of the heart E. Acute posterior infarction in the apex of the heart ANSWER: D 6. A ventricular depolarization wave when traveling 90 in the frontal plane will cause a large positive deflection in: A. aVR B. aVL C. Lead II
  • 46. ANSWER: A Questions 7-9: A 70-year-old man had the following EKG recorded at his doctor’s office. His weight was 80 kg and his blood pressure was 160/90 mmHg. He complained that he gets tired easily. 7. What is the mean electrical axis calculated from standard leads I, II, and III shown in the man’s EKG? A. −90 B. −60 C. −15 D. +60 E. +170 ANSWER: C 8. What is the heart rate using lead I for the calculation? A. 75 B. 88 C. 100 D. 112 E. 148 ANSWER : B 9. What is his likely diagnosis? A. Tricuspid valve stenosis B. Complete left bundle branch block C. Pulmonary valve stenosis D. Pulmonary valve regurgitation E. Left ventricular hypertrophy ANSWER: E 10. A seventy-year-old woman came into the local emergency room complaining of chest pain and shortness of breath. Blood samples were taken for cardiac enzyme measurements, and an EKG was quickly taken. Based on the EKG tracing shown below, what is the likely diagnosis? A. Left bundle branch block B. Acute anterior wall infarction in the left ventricle C. Acute anterior wall infarction in the right ventricle D. Acute posterior infarction in the left ventricle E. Acute posterior infarction in the right ventricle ANSWER: B
  • 47. Questions 11-12: A 65-year-old man went for an annual checkup and had the following EKG recorded at his physician's office. 11. What is the mean electrical axis calculated from standard leads I, II, and III shown in his EKG? A. −50 B. −15 C. 0 D. +15 E. +50 ANSWER: B 12. What is the most likely diagnosis? A. Right ventricular hypertrophy B. Left bundle branch block C. Pulmonary valve insufficiency D. Right bundle branch block E. Systemic hypertension ANSWER: E Questions 13-14. A 50-year-old woman was admitted to a local emergency room following a motorcycle accident, and the following EKG was taken. 13. What is her heart rate? Use lead I for the calculation. A. 56 B. 66 C. 76 D. 103 E. 152 ANSWER : D 14. What is the most likely diagnosis? A. Aortic valve insufficiency B. Left bundle branch block C. Pulmonary valve stenosis D. Right bundle branch block E. Systemic hypertension
  • 48. ANSWER: C 15. During the TP interval in an electrocardiogram of a patient with a damaged cardiac muscle: A. The entire heart is depolarized B. All the heart is depolarized except for the damaged cardiac muscle C. About half the heart is depolarized D. All of the heart is repolarized E. All of the heart is repolarized except for the damaged cardiac muscle ANSWER: E 16. A 30-year-old woman had an EKG taken, and her QRS complex had a width of 0.20 seconds, her T wave was inverted in lead I, and the R wave had a large negative deflection in lead III. What is her diagnosis? A. Right ventricular hypertrophy B. Left bundle branch block C. Pulmonary valve stenosis D. Right bundle branch block E. Left ventricular hypertrophy ANSWER: E 17. A ventricular depolarization wave when traveling −30 in the frontal plane will cause a large positive deflection in: A. aVR B. aVL C. Lead II D. Lead III E. aVF ANSWER: B Questions 18-19: A 6-year-old boy had the following EKG recorded at his physician's office. He had little energy, and during light exercise his respiratory would increase rapidly. 18. What is his mean electrical axis calculated from standard leads I, II, and III shown in his EKG? A. −100 B. −20 C. 0 D. +90 E. +170
  • 49. ANSWER: E 19. What is his diagnosis? A. Left ventricular hypertrophy B. Left bundle branch block C. Tricuspid stenosis D. Right bundle branch block E. Tetralogy of Fallot ANSWER: E E 20. Mr. Smith had his EKG measured at UMC, but they lost his records. The EKG technician remembered that the QRS deflection was large and negative in Lead aVF and 0 in Lead I. His mean electrical axis in the frontal plane is A. 90 B. 60 C. 0 D. −60 E. −90 ANSWER : E Chapter 13.Cardiac Arrhythmias and Their Electrocardiographic Interpretation 1. A forty-year-old man reported to his cardiologist that he had unusual feelings in his chest after drinking his fourth cup of coffee in the morning. The cardiologist recorded the following EKG during an exercise stress test. Leads II and III are shown. What is the likely diagnosis? A. Premature contraction originating in the atrium B. Premature contraction originating high in the A-V node C. Premature contraction originating low in the A-V node D. Premature contraction originating in the apex of the ventricle E. Premature contraction originating in the base of the ventricle ANSWER: EE
  • 50. 2. A ninety-year-old man had an EKG taken at his local doctor’s office, and the diagnosis was atrial fibrillation. Which of the following statements are likely conditions in someone with atrial fibrillation? A. Ventricular fibrillation normally accompanies atrial fibrillation B. The P waves of the EKG are strong C. The rate of ventricular contraction is irregular and fast D. The atrial “a” wave is normal E. The atria have a smaller volume than normal ANSWER: C 3. Circus movements in the ventricle can lead to ventricular fibrillation. Which of the following conditions in the ventricular muscle will increase the tendency for circus movements? A. Decreased refractory period B. Low extracellular potassium concentration C. Increased refractory period D. Shorter conduction pathway (decreased ventricular volume) E. Increase in parasympathetic impulses to the heart ANSWER: A 4. A 55-year-old woman had the following EKG recorded at her annual physical. She reported to her doctor that sometimes her pulse rate feels strong and at other times it feels weak. Standard limb lead II is shown. What is her likely diagnosis? A. Atrial paroxysmal tachycardia B. First degree A-V block C. Second degree A-V block, Mobitz type I (Wenckebach) D. Second degree A-V block, Mobitz type II E. Third degree A-V block ANSWER: D 5. A fifty-year-old man has been having fainting “spells” for about two weeks. During the fainting episodes, his EKG shows a ventricular beat of 25 per minute and 100 P waves per minute. After about 30 seconds of fainting a normal sinus rhythm reoccurs. What is his likely diagnosis? A. Atrial flutter B. First degree A-V block C. Second degree A-V block
  • 51. D. Third degree A-V block E. Stokes-Adams syndrome ANSWER: E 6. A 65-year-old woman had the following EKG recorded at her annual physical. She reported to her doctor that sometimes her pulse rate feels a little irregular. Standard limb lead III is shown. What is her likely diagnosis? A. Atrial paroxysmal tachycardia B. First degree A-V block C. Second degree A-V block, Mobitz type I (Wenckebach) D. Normal sinus rhythm E. Third degree A-V block ANSWER: EE 7. Circus movements in the ventricle can lead to ventricular fibrillation. Which of the following conditions in the ventricular muscle will increase the tendency for circus movements? A. Decreased conduction velocity B. Low extracellular potassium concentration C. Increased refractory period D. Shorter conduction pathway (decreased ventricular volume) E. Increase in parasympathetic impulses to the heart ANSWER: AA 8. Which of the following statements about a premature atrial contraction is true? A. The radial artery pulse associated with the premature contraction will be weak B. Stroke volume associated with the premature contraction will be increased C. The P wave is never seen D. The probability of these premature contractions occurring is decreased in people with a large caffeine intake E. It causes the QRS interval to be lengthened. ANSW ER: A A 9. A 65-year-old man reported some unusual sensations in his chest. His blood pressure is 135/85 mmHg, and he weighs 200 pounds. He had an EKG taken, and it is shown below. What is his likely diagnosis? A. Atrial paroxysmal tachycardia B. First degree A-V block C. Second degree A-V block D. Premature atrial contractions E. Premature ventricular contractions IF YOU WANT THIS TEST BANK OR SOLUTION MANUAL EMAIL ME
  • 52. rightmanforbloodline1@gmail.com TO RECEIVE ALL CHAPTERS IN PDF FORMAT IF YOU WANT THIS TEST BANK OR SOLUTION MANUAL EMAIL ME rightmanforbloodline1@gmail.com TO RECEIVE ALL CHAPTERS IN PDF FORMAT